Born's rule from almost nothing

357 views
Skip to first unread message

Philip Thrift

unread,
Dec 26, 2020, 8:48:07 AM12/26/20
to Everything List

Sabine Hossenfelder @skdh
Got an email tonight that my paper was accepted for publication. ...


Born's rule from almost nothing
Sabine Hossenfelder

Quantum mechanics does not make definite predictions but only predicts probabilities for measurement outcomes. One calculates these probabilities from the wave-function using Born’s rule. In axiomatic formulations of quantum mechanics, Born’s rule is usually added as an axiom on its own right. However, it seems the kind of assumption that should not require a postulate, but that should instead follow from the physical properties of the theory.

The argument discussed here is most similar to the ones for many worlds and the one using environment-assisted invariance. However, as will become clear shortly, the ontological baggage of these arguments is unnecessary.

Claim: The only well-defined and consistent distribution for transition probabilities on the complex sphere of dimension N which is continuous, independent of N, and invariant under unitary operations is [Born's rule]. The continuity assumption is unnecessary if one restricts the original space to states of norm K/N or, correspondingly, to rational-valued probabilities as a frequentist interpretation would suggest.


@philipthrift

Alan Grayson

unread,
Dec 26, 2020, 1:54:50 PM12/26/20
to Everything List
I think she believes in superdeterminism. Hence, IMO, not a reliable source. But I'll look at her paper anyway. AG

Lawrence Crowell

unread,
Dec 26, 2020, 7:17:50 PM12/26/20
to Everything List
It is a short paper. I will see what I can make of it in the near future. 

LC

Lawrence Crowell

unread,
Dec 27, 2020, 11:12:56 AM12/27/20
to Everything List
I read this and I have no quarrels with it. The only issue I might have is that it is more limited than a full Born rule. The only observable she works with is probability. This is then just a variant of Gleason's theorem. Sabine does not work with a general Hermitian operator or observable. However, the way she does this is similar to the Hilbert-Schmidt form and projective bundle. This might be worked into greater generality.

LC

On Saturday, December 26, 2020 at 7:48:07 AM UTC-6 cloud...@gmail.com wrote:

Philip Thrift

unread,
Dec 27, 2020, 1:42:20 PM12/27/20
to Everything List

Maybe she should have been a reviewer. ("I thank Scott Aaronson, Sandro Donadi, and Tim Palmer for helpful feedback.") As she tweets, this will be published in Annals of Physics.

This seems to be a fundamental result of what any probability distribution (under minimal assumptions) on a quantum-theoretic model must satisfy.

@philipthrift

Philip Thrift

unread,
Dec 27, 2020, 1:44:00 PM12/27/20
to Everything List
Correction:



Maybe you should have been a reviewer. ("I thank Scott Aaronson, Sandro Donadi, and Tim Palmer for helpful feedback.") As she tweets, this will be published in Annals of Physics.

Lawrence Crowell

unread,
Dec 27, 2020, 2:14:57 PM12/27/20
to Everything List
On Sunday, December 27, 2020 at 12:42:20 PM UTC-6 cloud...@gmail.com wrote:

Maybe she should have been a reviewer. ("I thank Scott Aaronson, Sandro Donadi, and Tim Palmer for helpful feedback.") As she tweets, this will be published in Annals of Physics.

This seems to be a fundamental result of what any probability distribution (under minimal assumptions) on a quantum-theoretic model must satisfy.

@philipthrift

That is a fair assessment, which makes this a step in the direction towards the Born rule. 

LC

Brent Meeker

unread,
Dec 27, 2020, 8:14:09 PM12/27/20
to everyth...@googlegroups.com
She implied that this proof was antithetical to the MWI, but I don't see how.

Brent
--
You received this message because you are subscribed to the Google Groups "Everything List" group.
To unsubscribe from this group and stop receiving emails from it, send an email to everything-li...@googlegroups.com.
To view this discussion on the web visit https://groups.google.com/d/msgid/everything-list/d054f397-c22a-4bd1-aec8-c881f851c1abn%40googlegroups.com.

Lawrence Crowell

unread,
Dec 28, 2020, 7:05:27 AM12/28/20
to Everything List
On Sunday, December 27, 2020 at 7:14:09 PM UTC-6 Brent wrote:
She implied that this proof was antithetical to the MWI, but I don't see how.

Brent


She says the proof is similar to Carroll and Sebens arXiv:1405.7907 [gr-qc]] without the "ontological baggage." I think the lack of this baggage means there is no explicit reference to MWI. I would say it is not so much ontological baggage but interpretation baggage that is discarded. 

LC

Bruno Marchal

unread,
Dec 29, 2020, 10:41:20 AM12/29/20
to everyth...@googlegroups.com
On 27 Dec 2020, at 17:12, Lawrence Crowell <goldenfield...@gmail.com> wrote:

I read this and I have no quarrels with it. The only issue I might have is that it is more limited than a full Born rule. The only observable she works with is probability. This is then just a variant of Gleason's theorem. Sabine does not work with a general Hermitian operator or observable. However, the way she does this is similar to the Hilbert-Schmidt form and projective bundle. This might be worked into greater generality.

I have the same feeling, but it is rather good, if not as general we could wish for. It might be better than the “usual” frequentist derivation (à-la Destouche-Février, who made a similar derivation (they were very good students of Louis de Broglie).

I read it quickly, some minor point are still a bit unclear, but the general idea is very nice.

Bruno





LC

On Saturday, December 26, 2020 at 7:48:07 AM UTC-6 cloud...@gmail.com wrote:

Sabine Hossenfelder @skdh
Got an email tonight that my paper was accepted for publication. ...


Born's rule from almost nothing
Sabine Hossenfelder

Quantum mechanics does not make definite predictions but only predicts probabilities for measurement outcomes. One calculates these probabilities from the wave-function using Born’s rule. In axiomatic formulations of quantum mechanics, Born’s rule is usually added as an axiom on its own right. However, it seems the kind of assumption that should not require a postulate, but that should instead follow from the physical properties of the theory.

The argument discussed here is most similar to the ones for many worlds and the one using environment-assisted invariance. However, as will become clear shortly, the ontological baggage of these arguments is unnecessary.

Claim: The only well-defined and consistent distribution for transition probabilities on the complex sphere of dimension N which is continuous, independent of N, and invariant under unitary operations is [Born's rule]. The continuity assumption is unnecessary if one restricts the original space to states of norm K/N or, correspondingly, to rational-valued probabilities as a frequentist interpretation would suggest.


@philipthrift

--
You received this message because you are subscribed to the Google Groups "Everything List" group.
To unsubscribe from this group and stop receiving emails from it, send an email to everything-li...@googlegroups.com.

Bruno Marchal

unread,
Dec 29, 2020, 10:47:05 AM12/29/20
to everyth...@googlegroups.com
On 28 Dec 2020, at 02:14, 'Brent Meeker' via Everything List <everyth...@googlegroups.com> wrote:

She implied that this proof was antithetical to the MWI, but I don't see how.

Does she? She said that it fits well with the MWI, but that she does not need that “metaphysical baggage” to get her proof. She is right, I think, but the MWI might still be the only way to make sense of this. She just seem to want to not use it. She says:

<<
The argument dis- cussed here is most similar to the ones for many worlds presented in [8, 9] and the one using environment-assisted invariance [7]. However, as will become clear shortly, the ontological baggage of these arguments is unnecessary.
>>

Bruno




Bruno Marchal

unread,
Dec 29, 2020, 10:51:24 AM12/29/20
to everyth...@googlegroups.com
On 28 Dec 2020, at 13:05, Lawrence Crowell <goldenfield...@gmail.com> wrote:

On Sunday, December 27, 2020 at 7:14:09 PM UTC-6 Brent wrote:
She implied that this proof was antithetical to the MWI, but I don't see how.

Brent


She says the proof is similar to Carroll and Sebens arXiv:1405.7907 [gr-qc]] without the "ontological baggage." I think the lack of this baggage means there is no explicit reference to MWI. I would say it is not so much ontological baggage but interpretation baggage that is discarded. 

Is not an interpretation always about the ontology of a theory. I would say that the MWI requires such type of derivation (of Born rule), but that such a derivation does not necessarily imply the MWI, as we can always add some magic to save an ontology. Her derivation might imply the MWI in the mechanist context, but that is not saying much, as mechanism implies the many computations from simple arithmetic.

Bruno



Lawrence Crowell

unread,
Dec 30, 2020, 5:48:34 PM12/30/20
to Everything List
On Tuesday, December 29, 2020 at 9:51:24 AM UTC-6 Bruno Marchal wrote:
On 28 Dec 2020, at 13:05, Lawrence Crowell <goldenfield...@gmail.com> wrote:

On Sunday, December 27, 2020 at 7:14:09 PM UTC-6 Brent wrote:
She implied that this proof was antithetical to the MWI, but I don't see how.

Brent


She says the proof is similar to Carroll and Sebens arXiv:1405.7907 [gr-qc]] without the "ontological baggage." I think the lack of this baggage means there is no explicit reference to MWI. I would say it is not so much ontological baggage but interpretation baggage that is discarded. 

Is not an interpretation always about the ontology of a theory. I would say that the MWI requires such type of derivation (of Born rule), but that such a derivation does not necessarily imply the MWI, as we can always add some magic to save an ontology. Her derivation might imply the MWI in the mechanist context, but that is not saying much, as mechanism implies the many computations from simple arithmetic.

Bruno


The two classes are interpretations that are ψ-ontological and ψ-epistemological. Interpretations such as Copenhagen Interpretation are epistemic. In a sense these carry "epistemic baggage."

LC

Bruce Kellett

unread,
Dec 31, 2020, 7:58:09 PM12/31/20
to Everything List
On Mon, Dec 28, 2020 at 11:05 PM Lawrence Crowell <goldenfield...@gmail.com> wrote:
On Sunday, December 27, 2020 at 7:14:09 PM UTC-6 Brent wrote:
She implied that this proof was antithetical to the MWI, but I don't see how.

Brent

She says the proof is similar to Carroll and Sebens arXiv:1405.7907 [gr-qc]] without the "ontological baggage." I think the lack of this baggage means there is no explicit reference to MWI. I would say it is not so much ontological baggage but interpretation baggage that is discarded.

Carroll and Sebens (and also Zurek) start with a Hilbert space vector with arbitrary coefficients weighting the basis vectors. They then expand the space so that all coefficients are equal. Hossenfelder takes a different route in that she starts with an N-dimensional Hilbert space with an orthonormal basis with equal coefficients, where each component has probability 1/N. She then forms a subspace from K of these basis vectors and shows that the vector spanning this subspace has probability K/N. From this you can clearly build up any arbitrary basis for an arbitrary dimensional space, and the basis vectors obtained will obey the Born Rule.

This approach is perhaps to be preferred over the arbitrary ad hoc expansion of the source space to get vectors of equal coefficients as done by Carroll/Sebens/Zurek. But it is still completely ad hoc in that it does not stem from any firmer basis than a desire to get the known right answer. After all, her starting assumption is that one wants probabilities from the theory, so she assumes a uniform probability distribution over her original Hilbert space. If one starts from there, getting the Born rule is essentially a triviality -- there are many routes, and Gleason's theorem is probably the most complicated. Simple appeals to Pythagoras in N dimensions for a normed vector space is probably all that is really necessary.

The trick, of course, is to justify the assumption of a probability distribution in the first place. One can appeal to experiment, and claim that the fact that it works is justification enough. But that fails to satisfy one's reductionist principles, and is equivalent to assuming Born's rule as an independent axiom, not in need of further justification. If one's claim, as made by MWI enthusiasts, is that the Schrodinger equation is all that one needs, taking the Born rule as an independent axiom does not work, and one needs to derive probabilities from the underlying deterministic theory. I claim that this is, of course, impossible. So the derivation of the Born rule still awaits a satisfactory answer.

Bruce

John Clark

unread,
Jan 1, 2021, 8:27:07 AM1/1/21
to everyth...@googlegroups.com
On Thu, Dec 31, 2020 at 7:58 PM Bruce Kellett <bhkel...@gmail.com> wrote:

> her starting assumption is that one wants probabilities from the theory,

That's not an assumption that's a goal, from experimentation we know what the correct probability is,  if a theory doesn't end up with that probability then that theory is wrong.

> it does not stem from any firmer basis than a desire to get the known right answer

Yes, but what's wrong with that? All theoreticians try to develop theories that conform with the right answer, aka experimental results. That's the entire point of science.
 
> so she assumes a uniform probability distribution over her original Hilbert space.

All theories have assumptions but you should always make the simplest assumptions, and since nobody has found a reason to think otherwise a uniform probability distribution is the simplest.

> The trick, of course, is to justify the assumption of a probability distribution in the first place. One can appeal to experiment, and claim that the fact that it works is justification enough. But that fails to satisfy one's reductionist principles,

Ultimately one's reductionist desires will always be frustrated. A new strange physical phenomenon is discovered, let's call it "A". Eventually somebody develops a brilliant new theory that says B causes A, but that leads to the question what causes B? Eventually somebody develops a brilliant new theory that says C causes B, but that leads to the question what causes C? There are only two possibilities, either this chain of questions goes on forever or it doesn't and it ends in a brute fact that has no cause, a hardcore reductionist would be unhappy with either outcome so I fear a hard-core reductionist is destined to be unhappy.

> is equivalent to assuming Born's rule as an independent axiom, not in need of further justification. If one's claim, as made by MWI enthusiasts, is that the Schrodinger equation is all that one needs, taking the Born rule as an independent axiom does not work, and one needs to derive probabilities from the underlying deterministic theory.

Assuming that Many Worlds is true and the multiverse is completely determined by Schrodinger's equation and there are therefore an astronomically large number (perhaps an infinite number) of Bruce Kelletts with microscopic or submicroscopic differences between them, and those Bruce Kelletts were observing a stream of photons polarized at angle X hit a polarizing filter set to angle X+Y; would any one of those Bruce Kelletts be able to predict with certainty that Bruce Kellett would or would not observe the photon pass through that filter? No. Would Bruce Kellett have to resort to probability? Yes. How would Bruce Kellett calculate the probability? If Bruce Kellett wanted to avoid logical self contradictions there is only one method Bruce Kellett could use, the Born Rule.

John K Clark    See my new list at  Extropolis

Brent Meeker

unread,
Jan 1, 2021, 5:35:08 PM1/1/21
to everyth...@googlegroups.com
I don't think that's quite true.  Suppose for example BK decided to predict that the polarization with the highest value of |psi|^2 is the one that would pass thru.  He wouldn't run into any logical contradiction because he's not interpreting it as probability, and so the fact that it doesn't provide a measure satisfying Kolomogorov's axioms is irrelevant.  And he wouldn't run into an empirical contradiction unless he assumed the actual process was producing a probability distribution and so he needed to predict a distribution and not just a value.  But then that's the point, one has to add that interpretive step to Schroedinger's equation.  Once you know that you need a probability distribution from the wave function...then Born's rule is the only choice.  But it's the step from the wave-function and "everything happens" to a probability distribution where MWI leaves a gap.

Brent


John K Clark    See my new list at  Extropolis
--
You received this message because you are subscribed to the Google Groups "Everything List" group.
To unsubscribe from this group and stop receiving emails from it, send an email to everything-li...@googlegroups.com.

Alan Grayson

unread,
Jan 1, 2021, 7:24:36 PM1/1/21
to Everything List
On Friday, January 1, 2021 at 3:35:08 PM UTC-7 Brent wrote:


On 1/1/2021 5:26 AM, John Clark wrote:
On Thu, Dec 31, 2020 at 7:58 PM Bruce Kellett <bhkel...@gmail.com> wrote:

> her starting assumption is that one wants probabilities from the theory,

That's not an assumption that's a goal, from experimentation we know what the correct probability is,  if a theory doesn't end up with that probability then that theory is wrong.

> it does not stem from any firmer basis than a desire to get the known right answer

Yes, but what's wrong with that? All theoreticians try to develop theories that conform with the right answer, aka experimental results. That's the entire point of science.
 
> so she assumes a uniform probability distribution over her original Hilbert space.

All theories have assumptions but you should always make the simplest assumptions, and since nobody has found a reason to think otherwise a uniform probability distribution is the simplest.

> The trick, of course, is to justify the assumption of a probability distribution in the first place. One can appeal to experiment, and claim that the fact that it works is justification enough. But that fails to satisfy one's reductionist principles,

Ultimately one's reductionist desires will always be frustrated. A new strange physical phenomenon is discovered, let's call it "A". Eventually somebody develops a brilliant new theory that says B causes A, but that leads to the question what causes B? Eventually somebody develops a brilliant new theory that says C causes B, but that leads to the question what causes C? There are only two possibilities, either this chain of questions goes on forever or it doesn't and it ends in a brute fact that has no cause, a hardcore reductionist would be unhappy with either outcome so I fear a hard-core reductionist is destined to be unhappy.

> is equivalent to assuming Born's rule as an independent axiom, not in need of further justification. If one's claim, as made by MWI enthusiasts, is that the Schrodinger equation is all that one needs, taking the Born rule as an independent axiom does not work, and one needs to derive probabilities from the underlying deterministic theory.

Assuming that Many Worlds is true and the multiverse is completely determined by Schrodinger's equation and there are therefore an astronomically large number (perhaps an infinite number) of Bruce Kelletts with microscopic or submicroscopic differences between them, and those Bruce Kelletts were observing a stream of photons polarized at angle X hit a polarizing filter set to angle X+Y; would any one of those Bruce Kelletts be able to predict with certainty that Bruce Kellett would or would not observe the photon pass through that filter? No. Would Bruce Kellett have to resort to probability? Yes. How would Bruce Kellett calculate the probability? If Bruce Kellett wanted to avoid logical self contradictions there is only one method Bruce Kellett could use, the Born Rule.

I don't think that's quite true.  Suppose for example BK decided to predict that the polarization with the highest value of |psi|^2 is the one that would pass thru.  He wouldn't run into any logical contradiction because he's not interpreting it as probability, and so the fact that it doesn't provide a measure satisfying Kolomogorov's axioms is irrelevant.  And he wouldn't run into an empirical contradiction unless he assumed the actual process was producing a probability distribution and so he needed to predict a distribution and not just a value.  But then that's the point, one has to add that interpretive step to Schroedinger's equation.  Once you know that you need a probability distribution from the wave function...then Born's rule is the only choice.  But it's the step from the wave-function and "everything happens" to a probability distribution where MWI leaves a gap.

Brent

Exactly. From the pov of the MWI, every trial is like another horse race where "everything happens" in those other worlds, but only ONE possible outcome per world. There's nothing in the interpretation which permits an ensemble of measurements in any of those worlds. On every trial a new set of worlds are created, and without an ensemble of measurements in any world, there can be no probabilities, and Born's rule is not applicable. I suppose the "gap" could be filled by adding another postulate to the MWI, but without it MWI fails with respect to satisfying any form of probability theory. AG

Alan Grayson

unread,
Jan 2, 2021, 12:10:02 AM1/2/21
to Everything List
The Guru from Australia seeks the origin of probability in QM. The answer is not hard to ascertain; the HUP. Heisenberg inferred the UP from physical principles, independent of QM, in his The Physical Principles of the Quantum Theory. You're welcome. AG 

Alan Grayson

unread,
Jan 2, 2021, 1:02:53 AM1/2/21
to Everything List
You can look at it this way; if the uncertainty in simultaneously knowing position and momentum is not due to lack of knowledge, that is due to ignorance, what are you left with? Answer; probability. AG 

John Clark

unread,
Jan 2, 2021, 4:17:12 AM1/2/21
to everyth...@googlegroups.com
On Fri, Jan 1, 2021 at 5:35 PM 'Brent Meeker' via Everything List <everyth...@googlegroups.com> wrote:

>> Assuming that Many Worlds is true and the multiverse is completely determined by Schrodinger's equation and there are therefore an astronomically large number (perhaps an infinite number) of Bruce Kelletts with microscopic or submicroscopic differences between them, and those Bruce Kelletts were observing a stream of photons polarized at angle X hit a polarizing filter set to angle X+Y; would any one of those Bruce Kelletts be able to predict with certainty that Bruce Kellett would or would not observe the photon pass through that filter? No. Would Bruce Kellett have to resort to probability? Yes. How would Bruce Kellett calculate the probability? If Bruce Kellett wanted to avoid logical self contradictions there is only one method Bruce Kellett could use, the Born Rule.

> I don't think that's quite true.  Suppose for example BK decided to predict that the polarization with the highest value of |psi|^2 is the one that would pass thru. He wouldn't run into any logical contradiction because he's not interpreting it as probability,

If the BKs are Interpreting that as a certainty and not a probability then the BKs wouldn't run into a logical contradiction but they would run into an empirical one because that wouldn't match experimental observation. It's entirely possible that a BK's prediction would fail and that the high |psi|^2 photon would NOT make it through (unless the value happened to be exactly 1), and even if the prediction turned out to be correct scientific experiments must be repeatable and when the BKs conduct it over and over again all the BKs will soon find out that the predictions tend to be correct |psi|^2 of the time.

 > he wouldn't run into an empirical contradiction unless he assumed the actual process was producing a probability distribution and so he needed to predict a distribution and not just a value. 

But the BKs didn't assume it was a probability distribution, they discovered it was. If the BKs assumed the |psi|^2 value was just a number and not a probability and had no physical significance then the BKs would soon discover that the assumption was wrong
  
> Once you know that you need a probability distribution from the wave function...then Born's rule is the only choice. 

Yes.
 
> But it's the step from the wave-function and "everything happens" to a probability distribution where MWI leaves a gap.

I don't see the gap. If Many Worlds was true then what would the Brent Meekers interpret |psi|^2 to mean? If it's just a number and means nothing then solving Schrodinger's equation would be a waste of time because that equation would also mean nothing, it should be ignored; but then we wouldn't have transistors or lasers or about 6.02*10^23 other things in modern life. 

Alan Grayson

unread,
Jan 2, 2021, 9:43:26 PM1/2/21
to Everything List
The gap Brent refers to has nothing to do with Schrodinger's equation, as I previously explained. Every trial in an experiment can be interpreted as a separate horse race, creating its own set of worlds where each possible occurrence is allegedly measured. But on subsequent trials, the MWI gives no guarantee that the same set of worlds is created. IOW, without another postulate appended to the MWI, each world is associated with exactly ONE measurement. No ensembles in these worlds; hence, the necessary condition for a probability doesn't exist. AG. 

John Clark

unread,
Jan 3, 2021, 4:42:26 AM1/3/21
to everyth...@googlegroups.com
On Sat, Jan 2, 2021 at 9:43 PM Alan Grayson <agrays...@gmail.com> wrote:

>> I don't see the gap. If Many Worlds was true then what would the Brent Meekers interpret |psi|^2 to mean? If it's just a number and means nothing then solving Schrodinger's equation would be a waste of time because that equation would also mean nothing, it should be ignored; but then we wouldn't have transistors or lasers or about 6.02*10^23 other things in modern life. 

> The gap Brent refers to has nothing to do with Schrodinger's equation, 
 
I don't know what you're talking about, the Schrodinger equation is where you get the psi function in the first place, and If that can't be used to obtain something observable, like a probability, then what's the point of the Schrodinger equation and why do physicists waste their time studying it ? We know from experiment that certainty is impossible in the quantum world, and we know from pure mathematics that if the Schrodinger equation Is meaningful and involves probability then that probability can only be |psi|^2. So assuming for the sake of argument that Many Worlds is true how would a logical version of Alan Grayson in one of those worlds interpret |psi|^2 to mean?
 
> Every trial in an experiment can be interpreted as a separate horse race,

No scientist that observes one race in which horse A beats horse B would conclude from that one observation that it is a general law of nature that horse A will always beat horse B! If a scientific experiment can't be repeated then it's not scientific. 

> creating its own set of worlds where each possible occurrence is allegedly measured.

But the scientists in each of those worlds conducting the experiment will have memories and records of having performed that experiment many many times before on many different photons and will observe that among the photons that made it through the polarizing filter most of them had a LARGE |psi|^2 value and that among the photons that did NOT make it through the polarizing filter most of them had a SMALL  |psi|^2 value. And from that all the nearly identical scientists in all those nearly identical worlds would conclude (and not assume) that |psi|^2 must be a probability.
 
> But on subsequent trials, the MWI gives no guarantee that the same set of worlds is created.

Obviously there are no predictions that are guaranteed to be correct, that's why you have to resort to probability. And until "you" actually observe the photon "you" can't be sure if "you" are in a world where the photon went through the filter or a world where the photon was absorbed by the filter; and even after "you" are  duplicated in one of Bruno's "you" duplicating machines "you" can't be sure if "you" are in Washington or Moscow until "you" open the door of the duplicating chamber and look out.  

> each world is associated with exactly ONE measurement

WRONG! As I said, each world has memories and records of having conducted the experiment many many times before.

Lawrence Crowell

unread,
Jan 3, 2021, 6:39:18 AM1/3/21
to Everything List
This is in part why I think these efforts are really variants of Gleason's theorem more than a proof of the Born rule. 

LC 

Alan Grayson

unread,
Jan 3, 2021, 10:31:34 AM1/3/21
to Everything List
I don't think you understand my point, which isn't complicated. On each trial in THIS world, different values are measured, like in a horse race run again and again with different winners. According to the MWI, in each trial many worlds are created depending on the possible outcomes. But there's nothing in the MWI which guarantees that the many worlds created on one trial, are identical to those created on any other trial. Hence, applying the MWI, the worlds allegedly created on each trial have only ONE measurement, not an ensemble of measurements as in THIS world, from which to infer probabilities. This has nothing to do with Schrodinger's equation even though it can be used to calculate probabilities. AG

John Clark

unread,
Jan 3, 2021, 2:11:19 PM1/3/21
to everyth...@googlegroups.com
On Sun, Jan 3, 2021 at 10:31 AM Alan Grayson <agrays...@gmail.com> wrote:
 
> On each trial in THIS world, different values are measured,

Yes, and each time a measurement in THIS world is made on a photon a different measurement in THAT world is made, but in both worlds the experimenters have memories and records of having done the same experiment many many times before, and when both experimenters look at their records they will find the photons that make it through the filter usually have a large |psi|^2 value and the photons that do NOT make it through the filter usually have a small |psi|^2 value. So both conclude that |psi|^2 Is a probability, and that has allowed quantum mechanics to make predictions that are accurate to better than one part in 1 billion.  
 
> But there's nothing in the MWI which guarantees that the many worlds created on one trial, are identical to those created on any other trial.

Huh? MWI  guarantees that unless a photon's  |psi|^2 value is zero or one in both worlds the measurements will never be identical, if they were there would be no split because for two worlds to diverge there must be a difference between one and another. If for example an unpolarized photon goes to a filter set to angle X and then passes through a second filter also set to X and then the probability it will make it through the second filter is 100%, and if the second filter was set to the angle X +90° the probability would be 0%; but If the second filter was set to any other angle probability would have to be used, for example if it was X +45° the probability would be 50-50. 
 
> Hence, applying the MWI, the worlds allegedly created on each trial have only ONE measurement,

Nobody can use probability if they only have one data point, fortunately all the experimenters in all the worlds have lots of them In their memories and in their records. 
 
> This has nothing to do with Schrodinger's equation even though it can be used to calculate probabilities. AG

That statement makes no sense. If you don't have Schrodinger's equation then you don't have psi, and if you don't have psi you can't use the Born Rule, and if you can't use the Born Rule you can't get probabilities, and in quantum mechanics if you don't have probabilities you don't have anything, and physics enters the dark ages  

Alan Grayson

unread,
Jan 3, 2021, 5:21:56 PM1/3/21
to Everything List
On Sunday, January 3, 2021 at 12:11:19 PM UTC-7 johnk...@gmail.com wrote:
On Sun, Jan 3, 2021 at 10:31 AM Alan Grayson <agrays...@gmail.com> wrote:
 
> On each trial in THIS world, different values are measured,

Yes, and each time a measurement in THIS world is made on a photon a different measurement in THAT world is made, but in both worlds the experimenters have memories and records of having done the same experiment many many times before, and when both experimenters look at their records they will find the photons that make it through the filter usually have a large |psi|^2 value and the photons that do NOT make it through the filter usually have a small |psi|^2 value. So both conclude that |psi|^2 Is a probability, and that has allowed quantum mechanics to make predictions that are accurate to better than one part in 1 billion.  
 
> But there's nothing in the MWI which guarantees that the many worlds created on one trial, are identical to those created on any other trial.

Huh? MWI  guarantees that unless a photon's  |psi|^2 value is zero or one in both worlds the measurements will never be identical, if they were there would be no split because for two worlds to diverge there must be a difference between one and another. If for example an unpolarized photon goes to a filter set to angle X and then passes through a second filter also set to X and then the probability it will make it through the second filter is 100%, and if the second filter was set to the angle X +90° the probability would be 0%; but If the second filter was set to any other angle probability would have to be used, for example if it was X +45° the probability would be 50-50. 

The MWI doesn't guarantee that these subsequent measurements, for subsequent horse races say, are occurring in the SAME OTHER worlds as trials progress, to get ensembles in those OTHER worlds. I find your arguments not dissimilar from those who believe the US elections of Nov 3 were rigged.  You seem to avoid the fact that no where does the MWI guarantee what's needed to get ensembles in those other worlds. You just affirm that it does. AG

John Clark

unread,
Jan 3, 2021, 5:56:51 PM1/3/21
to everyth...@googlegroups.com
On Sun, Jan 3, 2021 at 5:21 PM Alan Grayson <agrays...@gmail.com> wrote:

> The MWI doesn't guarantee that these subsequent measurements, for subsequent horse races say, are occurring in the SAME OTHER worlds as trials progress, to get ensembles in those OTHER worlds. 
 
I don't know what you mean by "SAME OTHER worlds", the same as what? In one world Alan Grayson remembers having seen the electron go left, in another world Alan Grayson remembers having seen the electron go right, other than that the two worlds are absolutely identical, so which one was the "SAME OTHER world"?

> You seem to avoid the fact that no where does the MWI guarantee [...]

Quantum mechanics is not in the guarantee business, it deals with probability.  

> I don't think you understand my point, which isn't complicated. 

Yes, your point is very simple indeed, but the word simple can have 2 meanings, one of them is complementary and the other not so much.  

Alan Grayson

unread,
Jan 3, 2021, 8:09:06 PM1/3/21
to Everything List
In first trial, the MWI postulates other worlds comes into existence. Same other worlds in second trial? Same other worlds in third trial, etc? Where does the MWI assert these other worlds are the SAME other worlds? Unless it does, you only have ONE measurement in each of these worlds. No probability exists in these other worlds since no ensemble of measurements exist in these other world. AG

John Clark

unread,
Jan 4, 2021, 7:08:00 AM1/4/21
to everyth...@googlegroups.com
On Sun, Jan 3, 2021 at 8:09 PM Alan Grayson <agrays...@gmail.com> wrote:

> In first trial, the MWI postulates other worlds comes into existence.

Yes.

> Same other worlds in second trial?

I don't understand the question. In the second trial just as in the first trial one Alan Grayson will see the electron go left and another Alan Grayson will see the electron go right, and neither of the second trial Alan Graysons will be the "same" as either of the first trial Alan Graysons. After a world splits it's meaningless to ask which world is the "same" world as the one before the split, the question makes no sense. All of the perhaps literally countless (it might be Aleph1) number of Alan Graysons will have memories and possess records that are different from each other; in some the differences will be tiny, submicroscopic, and others the differences will be huge, but In all cases whenever a Alan Grayson looks into the past he will see just one continuous chain of events that subjectively looks to him as one single world. And whenever an Alan Grayson looks to the future he will see absolutely nothing because none of the astronomical, and possibly infinite, number of Alan Graysons has memories or records from the future. That's the only reason the future is different from the past.

> Where does the MWI assert these other worlds are the SAME other worlds?

I don't know you tell me. Why would MWI even want to assert that "these other worlds are the SAME other worlds" ? I don't even know what that means.
 
> Unless it does, you only have ONE measurement in each of these worlds.

So.... you think inhabitants of such worlds don't have memories or records?!

Alan Grayson

unread,
Jan 4, 2021, 10:26:42 AM1/4/21
to Everything List
On Monday, January 4, 2021 at 5:08:00 AM UTC-7 johnk...@gmail.com wrote:
On Sun, Jan 3, 2021 at 8:09 PM Alan Grayson <agrays...@gmail.com> wrote:

> In first trial, the MWI postulates other worlds comes into existence.

Yes.

> Same other worlds in second trial?

I don't understand the question. In the second trial just as in the first trial one Alan Grayson will see the electron go left and another Alan Grayson will see the electron go right, and neither of the second trial Alan Graysons will be the "same" as either of the first trial Alan Graysons.

Right, but that's the problem. The other world AG in second trial has no memory of the other AG in first trial because they're in different worlds, unless you can show that the MWI puts both other AG's in the same other world. But the MWI doesn't do this unless you add another postulate to your MWI interpretation. AG

John Clark

unread,
Jan 4, 2021, 2:39:02 PM1/4/21
to everyth...@googlegroups.com
On Mon, Jan 4, 2021 at 10:26 AM Alan Grayson <agrays...@gmail.com> wrote:

 >> In the second trial just as in the first trial one Alan Grayson will see the electron go left and another Alan Grayson will see the electron go right, and neither of the second trial Alan Graysons will be the "same" as either of the first trial Alan Graysons.

> Right, but that's the problem. The other world AG in second trial has no memory of the other AG in first trial because they're in different worlds,
 
No. After  2 trials there are 4 Alan Graysons, 2 of them have the same records and memories for the first trial, both saw the electron go LEFT, but they have different memories for the second trial, one saw the electron go left and one saw the electron go right. And 2 other Alan Graysons also have the same records and memories for the first trial, both saw the electron go RIGHT, but they have different memories for the second trial, one saw the electron go left and the other saw the electron go right.  

There are 4 possibilities of how 2 coin flips could turn out they are HT,TH, HH and TT and if Many Worlds is correct then there is a Alan Grayson that has memories and records for each of them because in a coin flip bothheads and tails are outcomes that are consistent with the laws of physics, and everything that can happen does happen.

Alan Grayson

unread,
Jan 4, 2021, 5:40:21 PM1/4/21
to Everything List
Since the two flips are INDEPENDENT events, there's no reason to assume the AG CREATED in second flip has any memory of what the AG CREATED in first flip experienced. Only the AG in THIS world where we know both flips occurred, as the initiator of CREATED AG's, has a combination of memory and inferences of all occurrences. You can say anything, but that doesn't make it true, or implied by the MWI. AG

John Clark

unread,
Jan 5, 2021, 6:50:21 AM1/5/21
to everyth...@googlegroups.com
On Mon, Jan 4, 2021 at 5:40 PM Alan Grayson <agrays...@gmail.com> wrote:

>> There are 4 possibilities of how 2 coin flips could turn out they are HT,TH, HH and TT and if Many Worlds is correct then there is a Alan Grayson that has memories and records for each of them because in a coin flip bothheads and tails are outcomes that are consistent with the laws of physics, and everything that can happen does happen.

> Since the two flips are INDEPENDENT events, there's no reason to assume the AG CREATED in second flip has any memory of what the AG CREATED in first flip experienced.
 
After the first flip one Alan Grayson saw the electron go left (another Alan Grayson saw it go right but R Alan Grayson is not relevant right now) and L Alan Grayson performs the experiment again, and L Alan Grayson splits again, one L Alan Grayson sees the electron go left and one L Alan Grayson sees the electron go right, BUT both LL  Alan Grayson and LR Alan Grayson have identical memories and records about the first experiment, they both agree the electron went left, they disagree only about the second experiment.

Alan Grayson

unread,
Jan 5, 2021, 12:23:39 PM1/5/21
to Everything List
I don't see how your word salad answers my question. Physics is an empirical science and the observer in THIS world, who performs repeated trials, is never observed to "split". However, for the sake of argument, let's say another observer and world is CREATED and measures the opposite spin, assuming we're measuring spin along some axis, and everything that can happen, does happen. On the next trial, the observer in THIS world measures some spin and another world is created where the opposite spin is measured.  I don't see any necessary connection between these two CREATED worlds after two trials. What makes you think there is one? AG

John Clark

unread,
Jan 5, 2021, 12:27:45 PM1/5/21
to everyth...@googlegroups.com
On Tue, Jan 5, 2021 at 12:23 PM Alan Grayson <agrays...@gmail.com> wrote:

>> There are 4 possibilities of how 2 coin flips could turn out they are HT,TH, HH and TT and if Many Worlds is correct then there is a Alan Grayson that has memories and records for each of them because in a coin flip bothheads and tails are outcomes that are consistent with the laws of physics, and everything that can happen does happen.

> Since the two flips are INDEPENDENT events, there's no reason to assume the AG CREATED in second flip has any memory of what the AG CREATED in first flip experienced.
 
After the first flip one Alan Grayson saw the electron go left (another Alan Grayson saw it go right but R Alan Grayson is not relevant right now) and L Alan Grayson performs the experiment again, and L Alan Grayson splits again, one L Alan Grayson sees the electron go left and one L Alan Grayson sees the electron go right, BUT both LL  Alan Grayson and LR Alan Grayson have identical memories and records about the first experiment, they both agree the electron went left, they disagree only about the second experiment.

I don't see how your word salad answers my question.

You're hopeless. I give up.  

John K Clark


Alan Grayson

unread,
Jan 5, 2021, 12:45:40 PM1/5/21
to Everything List
Ditto. Until you detect a THIS WORLD observer splitting, you'll be on firm ground. Stop the Steal. AG 

John K Clark


John Clark

unread,
Jan 5, 2021, 12:55:47 PM1/5/21
to everyth...@googlegroups.com


On Tue, Jan 5, 2021 at 12:45 PM Alan Grayson <agrays...@gmail.com> wrote:

 > Until you detect a THIS WORLD observer splitting [....]

One world contains an Alan Grayson that sees the electron go left, another world is absolutely identical in every way except that it contains a  Alan Grayson that sees the electron go right. So you tell me, which of those 2 worlds is "THIS WORLD"?

John K Clark


Alan Grayson

unread,
Jan 5, 2021, 10:05:11 PM1/5/21
to Everything List
It's the world where a living being can observe the trials being measured. The other world is in your imagination (if you believe in the MWI). AG 

John K Clark


Alan Grayson

unread,
Jan 5, 2021, 11:04:55 PM1/5/21
to Everything List
FWIW, why, as someone with a strongly empirical interpretation of physics, do you give THIS world, the one we can directly observe, and the OTHER world, which is an imaginary construct, allegedly implied by mathematics,  equal ontological status? AG

John K Clark


smitra

unread,
Jan 6, 2021, 1:44:21 AM1/6/21
to everyth...@googlegroups.com
On 06-01-2021 05:04, Alan Grayson wrote:
> On Tuesday, January 5, 2021 at 8:05:11 PM UTC-7 Alan Grayson wrote:
>
>> On Tuesday, January 5, 2021 at 10:55:47 AM UTC-7 johnk...@gmail.com
>> wrote:
>>
>> On Tue, Jan 5, 2021 at 12:45 PM Alan Grayson <agrays...@gmail.com>
>> wrote:
>>
>>> _ __Until you detect a THIS WORLD observer splitting_ [....]
>>
>> One world contains an Alan Grayson that sees the electron go left,
>> another world is absolutely identical in every way except that it
>> contains a Alan Grayson that sees the electron go right. So you
>> tell me, which of those 2 worlds is "THIS WORLD"?
>
> It's the world where a living being can observe the trials being
> measured. The other world is in your imagination (if you believe in
> the MWI). AG
>
> FWIW, why, as someone with a strongly empirical interpretation of
> physics, do you give THIS world, the one we can directly observe, and
> the OTHER world, which is an imaginary construct, allegedly implied by
> mathematics, equal ontological status? AG


This World is selected at random from the set of all possible Worlds
using the Born rule probabilities. And we can infer that this is the
case from observations, although that doesn't rigorously prove that the
other Worlds are real. It's similar to inferring that the World as we
think it existed in 1985 did indeed exist, but we can never prove this,
because God could have created the World today with false memories about
a hypothetical 1985 that never really existed. There is no experiment we
can do to rigorously rule out that possibility. So, if there are
believers in the theory that God created the World yesterday, then there
is no experimental result available that will convince them that they
are wrong. The same is true about the MWI, albeit that the alternative
theories in that case are a bit less bizarre.

Saibal

John Clark

unread,
Jan 6, 2021, 5:33:52 AM1/6/21
to everyth...@googlegroups.com
On Tue, Jan 5, 2021 at 10:05 PM Alan Grayson <agrays...@gmail.com> wrote:

>> One world contains an Alan Grayson that sees the electron go left, another world is absolutely identical in every way except that it contains a  Alan Grayson that sees the electron go right. So you tell me, which of those 2 worlds is "THIS WORLD"?

> It's the world where a living being can observe the trials being measured. The other world is in your imagination (if you believe in the MWI). AG 

From that response I take it you have abandoned your attempt to poke logical holes in the Many Worlds Interpretation and instead have resorted to a pure emotional appeal; namely that there must be a fundamental law of physics that says anything Alan Grayson finds to be odd cannot exist, and Alan Grayson finds many Worlds to be odd. Personally I find Many Worlds to be odd too, although it's the least odd of all the quantum interpretations, however I don't think nature cares very much if you or I approve of it or not. From experimentation it's clear to me that if Many Worlds is not true then something even stranger is. 

 See my new list at  Extropolis

John K Clark


Alan Grayson

unread,
Jan 6, 2021, 6:41:57 AM1/6/21
to Everything List
I have no idea whatsoever, how you reached your conclusions above. There are things called laboratories, where physicists conduct experiments, some of which are quantum experiments with probabilistic outcomes. The world in which such things exist, I call THIS world. Worlds postulated to exist based on the claim that any possible measurement, must be a realized measurement in another world, I call OTHER worlds. Those OTHER worlds are imagined to exist based on the MWI. These are simple facts. I am not making any emotional appeals to anything. The possible oddness of the Cosmos is not affirmed or denied here. I agree the Cosmos might be odd, possibly very odd, but this has nothing to do with our discussion. The core of my argument is that since the trial outcomes in quantum experiments are independent of one another, there's no reason to claim that each of the OTHER worlds accumulates ensembles, as an ensemble is created in THIS world. Without ensembles in those OTHER worlds, the MWI fails to affirm the existence of probability in any of those OTHER worlds. AG 

Alan Grayson

unread,
Jan 6, 2021, 7:01:14 AM1/6/21
to Everything List
I should have been more explicit; since the trials are independent, the other worlds implied by the MWI for any particular trial, are unrelated to the other worlds created for any OTHER particular trial. Thus, each other world has an ensemble with one element, insufficient for the existence of probabilities. AG

Quentin Anciaux

unread,
Jan 6, 2021, 7:28:51 AM1/6/21
to everyth...@googlegroups.com
Here a schema:
image.png

After 3 experiments, you have *8* worlds... each with the memory of the initial experiment, 4 of the 2nd version A and for of the 2nd version B... etc

Every *worlds* has a past which is linked directly with the previous experiment and to the initial experiment... in each world there is an ensemble of 3 results.

Quentin

--
You received this message because you are subscribed to the Google Groups "Everything List" group.
To unsubscribe from this group and stop receiving emails from it, send an email to everything-li...@googlegroups.com.


--
All those moments will be lost in time, like tears in rain. (Roy Batty/Rutger Hauer)

John Clark

unread,
Jan 6, 2021, 7:35:44 AM1/6/21
to everyth...@googlegroups.com
On Wed, Jan 6, 2021 at 6:42 AM Alan Grayson <agrays...@gmail.com> wrote:

> There are things called laboratories, where physicists conduct experiments, some of which are quantum experiments with probabilistic outcomes.

If Many Worlds is correct then there are an astronomically large number, and possibly an infinitely large number, of worlds where physicists conduct experiments, some of which are quantum experiments with probabilistic outcomes.
 
The world in which such things exist, I call THIS world.

So there are an astronomically large number and possibly an infinitely large number of "THIS" worlds.

> Worlds postulated to exist based on the claim that any possible measurement, must be a realized measurement in another world, I call OTHER worlds.

Alan Grayson decides that tomorrow Alan Grayson will conduct an experiment to determine if an electron goes left or right. If Many Worlds is correct then the day after tomorrow one Alan Grayson will remember having seen the electron go left and one Alan Grayson will remember having seen the electron go right. Which Alan Grayson lives in "THIS" world.

> The core of my argument is that since the trial outcomes in quantum experiments are independent of one another, there's no reason to claim that each of the OTHER worlds accumulates ensembles, as an ensemble is created in THIS world.

That is just untrue. When one Alan Grayson has observed 1000 photons there is another Alan Grayson that agrees with 999 of the observations and disagrees only about #1000. All the 2^1000 Alan Graysons have made1000 observations, most Alan Graysons saw the electron go left about 500 times and go right about 500 times, but a few were quite different, one Alan Grayson out of  2^1000  saw the electron go left 1000 times in a row and one Alan Grayson out of 2^1000 saw the electron go right 1000 times in a row.

Alan Grayson

unread,
Jan 6, 2021, 7:55:16 AM1/6/21
to Everything List
If you don't know what THIS world is, I can't help. AG 

sce...@libero.it

unread,
Jan 6, 2021, 8:17:18 AM1/6/21
to everyth...@googlegroups.com

Worlds, worlds. What are these worlds? When a pig observes a Young interferometer does this pig create worlds? Does this pig split worlds? Or not, because there is not full consciousness? And in Alpha Centauri,  where there are no pigs, no humans, no consciousness, no Young interferometers? No Franson interferometers either ...

--
Inviato da Libero Mail per Android

Mercoledì, 06 Gennaio 2021, 01:28PM +01:00 da Quentin Anciaux allc...@gmail.com:

Quentin Anciaux

unread,
Jan 6, 2021, 8:17:35 AM1/6/21
to everyth...@googlegroups.com
There are no this world... Every world is a world with a past. To simplify we will assume with start with one world at t, at t1, we have two worlds, at t2, 4 and so on... Each of these worlds are direct continuation of the unique world at time t, at t3, there is no *this worlds*, every alan in each worlds point to it saying this world... But there is no This world, it makes no sense.

Quentin

--
You received this message because you are subscribed to the Google Groups "Everything List" group.
To unsubscribe from this group and stop receiving emails from it, send an email to everything-li...@googlegroups.com.

Quentin Anciaux

unread,
Jan 6, 2021, 8:19:34 AM1/6/21
to everyth...@googlegroups.com
I think there is no split, but continuous differentiation. So there is always an infinity of worlds. Or there is no world at all and only consciousness differentiation.

Quentin

noname
noname

John Clark

unread,
Jan 6, 2021, 8:39:11 AM1/6/21
to everyth...@googlegroups.com
On Wed, Jan 6, 2021 at 7:55 AM Alan Grayson <agrays...@gmail.com> wrote:

>> Alan Grayson decides that tomorrow Alan Grayson will conduct an experiment to determine if an electron goes left or right. If Many Worlds is correct then the day after tomorrow one Alan Grayson will remember having seen the electron go left and one Alan Grayson will remember having seen the electron go right. Which Alan Grayson lives in "THIS" world?

> If you don't know what THIS world is, I can't help. AG 

Yes I know you can't help, I asked a simple question and you were unable to answer it

Alan Grayson

unread,
Jan 6, 2021, 8:49:16 AM1/6/21
to Everything List
You've drank too much kool-aid to understand my answer. AG 

Terren Suydam

unread,
Jan 6, 2021, 8:50:29 AM1/6/21
to Everything List
This is how I see it as well. All possible worlds already exist in a platonic sense, and one's experience represents a single path traversed through the infinite multitude of possibilities. This connects nicely to the universal dovetailer idea.

Brent Meeker

unread,
Jan 6, 2021, 4:12:51 PM1/6/21
to everyth...@googlegroups.com
Julian Barbour had a metaphor in which the multiverse is like a river with infinitely many streamlines which differ only at a microscopic level where they can interfere but it makes no difference at the macroscopic level of the river.  But there can be divisions by islands in the stream that split the river into macroscopically different rivers with different destinations.  The split isn't absolute.  The islands are permeable.  But statistically they are almost absolute. 

I don't agree that consciousness is the differentiation.  The problem with that view is that there are many macroscopic splits which are not noticed by any consciousness.  But may be inferred much later.   Sure it takes consciousness (or some kind) to infer them later, but that inference fails to produce an understandable and predictive picture if the split is not placed in the past...and evolution produced consciousness in order to provide understandable and predictive pictures.

Brent

Philip Benjamin

unread,
Jan 7, 2021, 11:15:17 AM1/7/21
to everyth...@googlegroups.com

[Brent Meeker]

“Julian Barbour had a metaphor in which the multiverse is like a river with infinitely many streamlines…… evolution produced consciousness in order to provide understandable and predictive pictures”.

[Philip Benjamin]

    The phrase “in order to provide” is ‘consciously’ used here, buy simply connotes purpose and meaning which are foreign to evolutionary notions of randomness devoid of direction. The whole idea of multiverse is contrary to all laws of logic and the fundamental physical law of conservation of mass and energy. Matter-wave duality is a misrepresentation of particles behaving AS IF waveforms. What is needed here is AS IF logic not Both & fallacy.  Furthermore “multiverse” will require a multiverse chemistry to be meaningful in anyway, especially for any biosphere. Bio dark-matter with bio dark-matter chemistry (chemical bonds or spin governed duets and octets) with

biophoton emissions as evidence of dark and light chemical bond interactions is more reasonable.          

     Augustine’s consciousness was instantly transformed form paganism to non-paganism  (https://www.midwestaugustinians.org/conversion-of-st-augustine). He considered Plato as a great thinker who "understood the eternal truth" consistent with Biblical ideology. Plato's metaphysics and epistemology molded Augustine's understanding of the Biblical Deity as a source of absolute goodness and truth. This idea mirrored Plato's thinking of "forms", where every object in the universe is a representation of a perfect idea of that entity--  a tree is an imperfect version of a perfect form of a tree. Augustine “baptized” Platonism into Scriptural norms and thus pulled the West form the Greco-Roman pagan superstitions. For Augustine the Biblical God is the source of the forms. Who is the Biblical God, other than Adonai (plural) YHWH (singular) Elohim (uni-plural)? That is an impossible concept for un-Augustinian Western Acade-Media Pagan(ism)—WAMP-the-Ingrate.  Augustine concluded that what is really beautiful cannot be seen with our eyes at all. Knowledge of beauty is richer than its sight, it is the knowledge of the very source and creator of all beauty, who is God Himself, through light of faith and virtue (Confessions, Trans. Gary Wills, Penguin Classics; Deluxe Edition, January 31, 2006 as quoted at https://intellectualcharity.org/2013/10/27/augustine-on-beauty/).

Philip Benjamin

 

From: 'Brent Meeker' via Everything List <everyth...@googlegroups.com>
Sent: Wednesday, January 6, 2021 3:13 PM
To: everyth...@googlegroups.com
Subject: Re: Born's rule from almost nothing

 

Julian Barbour had a metaphor in which the multiverse is like a river with infinitely many streamlines which differ only at a microscopic level where they can interfere but it makes no difference at the macroscopic level of the river.  But there can be divisions by islands in the stream that split the river into macroscopically different rivers with different destinations.  The split isn't absolute.  The islands are permeable.  But statistically they are almost absolute. 

I don't agree that consciousness is the differentiation.  The problem with that view is that there are many macroscopic splits which are not noticed by any consciousness.  But may be inferred much later.   Sure it takes consciousness (or some kind) to infer them later, but that inference fails to produce an understandable and predictive picture if the split is not placed in the past...and evolution produced consciousness in order to provide understandable and predictive pictures.

Brent

On 1/6/2021 5:19 AM, Quentin Anciaux wrote:

I think there is no split, but continuous differentiation. So there is always an infinity of worlds. Or there is no world at all and only consciousness differentiation.

 

Quentin

 

Le mer. 6 janv. 2021 à 14:17, scerir via Everything List <everyth...@googlegroups.com> a écrit :

Worlds, worlds. What are these worlds? When a pig observes a Young interferometer does this pig create worlds? Does this pig split worlds? Or not, because there is not full consciousness? And in Alpha Centauri,  where there are no pigs, no humans, no consciousness, no Young interferometers? No Franson interferometers either ...

--
Inviato da Libero Mail per Android

Mercoledì, 06 Gennaio 2021, 01:28PM +01:00 da Quentin Anciaux allc...@gmail.com:


Here a schema:

Alan Grayson

unread,
Jan 7, 2021, 9:32:00 PM1/7/21
to Everything List
On Wednesday, January 6, 2021 at 6:39:11 AM UTC-7 johnk...@gmail.com wrote:
This is why you seem to "argue" like a Trumper (even though you're not). You asked me to define "THIS world" and I gave you a straight forward answer, which inexplicably you can't seem to understand. It's the world where anyone one of us can witness physicists measuring a quantum event, or using instruments for the measuring. It is from THIS world that all your hypothetical worlds derive or are implied by the mathematics of quantum physics, applying the MWI. No one can actually witness any measurements in, say, the initial other world, created as a result of the first trial in THIS world. You claim to be an empiricist when it comes to physics. Why do you dramatically depart from this POV or philosophy when it comes to the MWI? Baffling. AG

John Clark

unread,
Jan 8, 2021, 9:19:35 AM1/8/21
to everyth...@googlegroups.com
On Thu, Jan 7, 2021 at 9:32 PM Alan Grayson <agrays...@gmail.com> wrote:

> This is why you seem to "argue" like a Trumper (even though you're not). You asked me to define "THIS world" and I gave you a straight forward answer, which inexplicably you can't seem to understand.

It's perfectly OK to use the word "this" In everyday conversation, but what we were discussing is if the idea that "THIS world" Is always splitting many trillions of times a second is valid or not. In that context the meaning of the phrase "THIS world" is far from obvious.
 
> It's the world where anyone one of us can witness physicists measuring a quantum event,

Any ONE? If Many Worlds is correct then an astronomical number of Alan Graysons remember witnessing a physicist measure a quantum event, but the Alan Graysons will not all have the same memories.
  John K Clark      See my new list at  Extropolis

John Clark

unread,
Jan 8, 2021, 9:59:41 AM1/8/21
to everyth...@googlegroups.com
On Thu, Jan 7, 2021 at 11:15 AM Philip Benjamin <medin...@hotmail.com> wrote:

> The whole idea of multiverse is contrary to all laws of logic

My hunch is that Many Worlds Is correct but I could be wrong, but right or wrong it doesn't violate any law of logic, it's odd but not paradoxical and we already know from experiment that whatever the true interpretation of Quantum Mechanics turns out to be it's going to be odd, very very odd.
 

 and the fundamental physical law of conservation of mass and energy.

 
Unlike the Second Law Of Thermodynamics which we know from pure logic must be true a violation in the law of conservation of mass and energy would not violate any laws of logic. And even before Quantum Mechanics came along we knew from General Relativity that the law of conservation of mass-energy only holds true locally not globally. The universe is expanding so photons of light get stretched to a longer wavelength and photons with a longer wavelength have less energy, so where did that energy go? It didn't go anywhere, it just disappeared. And now we know that the universe is accelerating, so where did the energy for that acceleration come from? It came from empty space. It's still true that if a spherical region of space of fixed size is in thermal equilibrium then the amount of energy entering that sphere will equal the amount of energy leaving it, it's just that now more space has been created outside that sphere and thus there is more energy.  

 > Augustine’s consciousness was instantly transformed form paganism to non-paganism


Personally I don't think the change from polytheism to monotheism was much of an advance, both ideas are silly but polytheism is more fun, it makes for better comic books. 
John K Clark    See my new list at Extropolis

Alan Grayson

unread,
Jan 9, 2021, 12:25:56 AM1/9/21
to Everything List
Let's say AG goes into a lab and does repeated quantum spin measurements along some axis. THIS AG will have memories of ALL his observations and he's NEVER observed being split. Moreover, if we accept the MWI, those OTHER worlds and OTHER AG's, will be disjoint from each other, and will have the memory of only ONE observation. AG

John Clark

unread,
Jan 9, 2021, 6:52:39 AM1/9/21
to everyth...@googlegroups.com
On Sat, Jan 9, 2021 at 12:25 AM Alan Grayson <agrays...@gmail.com> wrote:

> Let's say AG goes into a lab and does repeated quantum spin measurements along some axis. THIS AG will have memories of ALL his observations

Yes, and if the Many Worlds Interpretation is correct then while Alan Grayson is making those quantum spin measurements Alan Grayson is splitting many trillions of times a second and has been doing so every second since Alan Grayson was born. So which  Alan Grayson is "THIS Alan Grayson" ?

> and OTHER AG's, will be disjoint from each other

I don't know what that means. How would things be different if the other Alan Graysons were un-disjointed?

Alan Grayson

unread,
Jan 9, 2021, 8:32:59 AM1/9/21
to Everything List
On Saturday, January 9, 2021 at 4:52:39 AM UTC-7 johnk...@gmail.com wrote:
On Sat, Jan 9, 2021 at 12:25 AM Alan Grayson <agrays...@gmail.com> wrote:

> Let's say AG goes into a lab and does repeated quantum spin measurements along some axis. THIS AG will have memories of ALL his observations

Yes, and if the Many Worlds Interpretation is correct then while Alan Grayson is making those quantum spin measurements Alan Grayson is splitting many trillions of times a second and has been doing so every second since Alan Grayson was born. So which  Alan Grayson is "THIS Alan Grayson" ?

And the empirical evidence for this is what? Is the tooth fairy also splitting? AG 

> and OTHER AG's, will be disjoint from each other

I don't know what that means. How would things be different if the other Alan Graysons were un-disjointed?

"Disjointed" means no contact between the AG's which allegedly exist due to the "splits". What evidence is there that any of these AG's have any contact with each other?  AG

John Clark

unread,
Jan 9, 2021, 10:04:39 AM1/9/21
to everyth...@googlegroups.com
On Sat, Jan 9, 2021 at 8:33 AM Alan Grayson <agrays...@gmail.com> wrote:


>> Yes, and if the Many Worlds Interpretation is correct then while Alan Grayson is making those quantum spin measurements Alan Grayson is splitting many trillions of times a second and has been doing so every second since Alan Grayson was born. So which  Alan Grayson is "THIS Alan Grayson" ?

>And the empirical evidence for this is what?

The double slit experiment. It's not proof but it is strong evidence.  
 
> Is the tooth fairy also splitting? AG 

I get it, Alan Grayson finds Many Worlds to be odd,  I do too, but if Many Worlds is not the correct quantum interpretation then something even stranger is .

>>> and OTHER AG's, will be disjoint from each other

>> I don't know what that means. How would things be different if the other Alan Graysons were un-disjointed?

> "Disjointed" means no contact between the AG's

There will be no future contact with the Alan Graysons but there was obviously past contact because they both have identical memories up to the Instant of the split. 

John K Clark      See my new list at  Extropolis 
 
You received this message because you are subscribed to the Google Groups "Everything List" group.

To unsubscribe from this group and stop receiving emails from it, send an email to everything-li...@googlegroups.com.

Alan Grayson

unread,
Jan 9, 2021, 10:35:08 AM1/9/21
to Everything List
On Saturday, January 9, 2021 at 8:04:39 AM UTC-7 johnk...@gmail.com wrote:
On Sat, Jan 9, 2021 at 8:33 AM Alan Grayson <agrays...@gmail.com> wrote:


>> Yes, and if the Many Worlds Interpretation is correct then while Alan Grayson is making those quantum spin measurements Alan Grayson is splitting many trillions of times a second and has been doing so every second since Alan Grayson was born. So which  Alan Grayson is "THIS Alan Grayson" ?

>And the empirical evidence for this is what?

The double slit experiment. It's not proof but it is strong evidence.  

What evidence are you referring to? AG 
 
> Is the tooth fairy also splitting? AG 

I get it, Alan Grayson finds Many Worlds to be odd,  I do too, but if Many Worlds is not the correct quantum interpretation then something even stranger is .

I told you previously. It's not that it's odd that bothers me; it's that it's grasping at straws to explain something we don't understand. And it's based on a deep, non-empirically supported fallacy; that, for example, in a horse race, there MUST be worlds in which every horse wins. Also, the MWI enthusiasts seem oblivious to the issue of where the energy comes from to create these other worlds. The interpretation is not simply odd; rather it seems dumb, based on a dubious hypothesis and ignoring the NON-EXISTENCE OF EVIDENCE that those other worlds even exist.  AG 

>>> and OTHER AG's, will be disjoint from each other

>> I don't know what that means. How would things be different if the other Alan Graysons were un-disjointed?

> "Disjointed" means no contact between the AG's

There will be no future contact with the Alan Graysons but there was obviously past contact because they both have identical memories up to the Instant of the split. 

You refuse to understand a point I have made repeatedly throughout. There's no reason to assume that any OTHER AG's are in contact with each other. Maybe you need to add another hypothesis to your MWI. AG 

Bruno Marchal

unread,
Jan 12, 2021, 7:26:50 AM1/12/21
to everyth...@googlegroups.com

On 3 Jan 2021, at 03:43, Alan Grayson <agrays...@gmail.com> wrote:



On Saturday, January 2, 2021 at 2:17:12 AM UTC-7 johnk...@gmail.com wrote:
On Fri, Jan 1, 2021 at 5:35 PM 'Brent Meeker' via Everything List <everyth...@googlegroups.com> wrote:

>> Assuming that Many Worlds is true and the multiverse is completely determined by Schrodinger's equation and there are therefore an astronomically large number (perhaps an infinite number) of Bruce Kelletts with microscopic or submicroscopic differences between them, and those Bruce Kelletts were observing a stream of photons polarized at angle X hit a polarizing filter set to angle X+Y; would any one of those Bruce Kelletts be able to predict with certainty that Bruce Kellett would or would not observe the photon pass through that filter? No. Would Bruce Kellett have to resort to probability? Yes. How would Bruce Kellett calculate the probability? If Bruce Kellett wanted to avoid logical self contradictions there is only one method Bruce Kellett could use, the Born Rule.

> I don't think that's quite true.  Suppose for example BK decided to predict that the polarization with the highest value of |psi|^2 is the one that would pass thru. He wouldn't run into any logical contradiction because he's not interpreting it as probability,

If the BKs are Interpreting that as a certainty and not a probability then the BKs wouldn't run into a logical contradiction but they would run into an empirical one because that wouldn't match experimental observation. It's entirely possible that a BK's prediction would fail and that the high |psi|^2 photon would NOT make it through (unless the value happened to be exactly 1), and even if the prediction turned out to be correct scientific experiments must be repeatable and when the BKs conduct it over and over again all the BKs will soon find out that the predictions tend to be correct |psi|^2 of the time.

 > he wouldn't run into an empirical contradiction unless he assumed the actual process was producing a probability distribution and so he needed to predict a distribution and not just a value. 

But the BKs didn't assume it was a probability distribution, they discovered it was. If the BKs assumed the |psi|^2 value was just a number and not a probability and had no physical significance then the BKs would soon discover that the assumption was wrong
  
> Once you know that you need a probability distribution from the wave function...then Born's rule is the only choice. 

Yes.
 
> But it's the step from the wave-function and "everything happens" to a probability distribution where MWI leaves a gap.

I don't see the gap. If Many Worlds was true then what would the Brent Meekers interpret |psi|^2 to mean? If it's just a number and means nothing then solving Schrodinger's equation would be a waste of time because that equation would also mean nothing, it should be ignored; but then we wouldn't have transistors or lasers or about 6.02*10^23 other things in modern life. 

The gap Brent refers to has nothing to do with Schrodinger's equation, as I previously explained. Every trial in an experiment can be interpreted as a separate horse race, creating its own set of worlds where each possible occurrence is allegedly measured. But on subsequent trials, the MWI gives no guarantee that the same set of worlds is created. IOW, without another postulate appended to the MWI, each world is associated with exactly ONE measurement. No ensembles in these worlds; hence, the necessary condition for a probability doesn't exist. AG. 

The born rule must be applied, and it concerns the relative accessible histories. It is better to avoid the term “world” which is hard to define.

Bruno




John K Clark   See my new list at  Extropolis
 

--
You received this message because you are subscribed to the Google Groups "Everything List" group.
To unsubscribe from this group and stop receiving emails from it, send an email to everything-li...@googlegroups.com.

Bruno Marchal

unread,
Jan 12, 2021, 7:32:23 AM1/12/21
to everyth...@googlegroups.com

On 30 Dec 2020, at 23:48, Lawrence Crowell <goldenfield...@gmail.com> wrote:

On Tuesday, December 29, 2020 at 9:51:24 AM UTC-6 Bruno Marchal wrote:
On 28 Dec 2020, at 13:05, Lawrence Crowell <goldenfield...@gmail.com> wrote:

On Sunday, December 27, 2020 at 7:14:09 PM UTC-6 Brent wrote:
She implied that this proof was antithetical to the MWI, but I don't see how.

Brent


She says the proof is similar to Carroll and Sebens arXiv:1405.7907 [gr-qc]] without the "ontological baggage." I think the lack of this baggage means there is no explicit reference to MWI. I would say it is not so much ontological baggage but interpretation baggage that is discarded. 

Is not an interpretation always about the ontology of a theory. I would say that the MWI requires such type of derivation (of Born rule), but that such a derivation does not necessarily imply the MWI, as we can always add some magic to save an ontology. Her derivation might imply the MWI in the mechanist context, but that is not saying much, as mechanism implies the many computations from simple arithmetic.

Bruno


The two classes are interpretations that are ψ-ontological and ψ-epistemological. Interpretations such as Copenhagen Interpretation are epistemic. In a sense these carry "epistemic baggage.”

At some point I think we should formalise our theory, because if terms like “epistemic” and “ontological” have sense only relative to some metaphysics. With mechanism, the entire physical reality is a projection of infinitely many number relations. This extends Everett, as it makes the wave emerging from the computational histories (which belong to the arithmetical reality, as we know since the 1930s).

My feeling is that you are assuming some ontological physical universe, but I can explain that this makes no sense once we assume even just the minimal amount of digital mechanism to attribute an explanative power to Darwin’s theory of evolution (say).

Bruno





LC

--
You received this message because you are subscribed to the Google Groups "Everything List" group.
To unsubscribe from this group and stop receiving emails from it, send an email to everything-li...@googlegroups.com.

Alan Grayson

unread,
Jan 12, 2021, 7:41:07 AM1/12/21
to Everything List
On Tuesday, January 12, 2021 at 5:26:50 AM UTC-7 Bruno Marchal wrote:

On 3 Jan 2021, at 03:43, Alan Grayson <agrays...@gmail.com> wrote:



On Saturday, January 2, 2021 at 2:17:12 AM UTC-7 johnk...@gmail.com wrote:
On Fri, Jan 1, 2021 at 5:35 PM 'Brent Meeker' via Everything List <everyth...@googlegroups.com> wrote:

>> Assuming that Many Worlds is true and the multiverse is completely determined by Schrodinger's equation and there are therefore an astronomically large number (perhaps an infinite number) of Bruce Kelletts with microscopic or submicroscopic differences between them, and those Bruce Kelletts were observing a stream of photons polarized at angle X hit a polarizing filter set to angle X+Y; would any one of those Bruce Kelletts be able to predict with certainty that Bruce Kellett would or would not observe the photon pass through that filter? No. Would Bruce Kellett have to resort to probability? Yes. How would Bruce Kellett calculate the probability? If Bruce Kellett wanted to avoid logical self contradictions there is only one method Bruce Kellett could use, the Born Rule.

> I don't think that's quite true.  Suppose for example BK decided to predict that the polarization with the highest value of |psi|^2 is the one that would pass thru. He wouldn't run into any logical contradiction because he's not interpreting it as probability,

If the BKs are Interpreting that as a certainty and not a probability then the BKs wouldn't run into a logical contradiction but they would run into an empirical one because that wouldn't match experimental observation. It's entirely possible that a BK's prediction would fail and that the high |psi|^2 photon would NOT make it through (unless the value happened to be exactly 1), and even if the prediction turned out to be correct scientific experiments must be repeatable and when the BKs conduct it over and over again all the BKs will soon find out that the predictions tend to be correct |psi|^2 of the time.

 > he wouldn't run into an empirical contradiction unless he assumed the actual process was producing a probability distribution and so he needed to predict a distribution and not just a value. 

But the BKs didn't assume it was a probability distribution, they discovered it was. If the BKs assumed the |psi|^2 value was just a number and not a probability and had no physical significance then the BKs would soon discover that the assumption was wrong
  
> Once you know that you need a probability distribution from the wave function...then Born's rule is the only choice. 

Yes.
 
> But it's the step from the wave-function and "everything happens" to a probability distribution where MWI leaves a gap.

I don't see the gap. If Many Worlds was true then what would the Brent Meekers interpret |psi|^2 to mean? If it's just a number and means nothing then solving Schrodinger's equation would be a waste of time because that equation would also mean nothing, it should be ignored; but then we wouldn't have transistors or lasers or about 6.02*10^23 other things in modern life. 

The gap Brent refers to has nothing to do with Schrodinger's equation, as I previously explained. Every trial in an experiment can be interpreted as a separate horse race, creating its own set of worlds where each possible occurrence is allegedly measured. But on subsequent trials, the MWI gives no guarantee that the same set of worlds is created. IOW, without another postulate appended to the MWI, each world is associated with exactly ONE measurement. No ensembles in these worlds; hence, the necessary condition for a probability doesn't exist. AG. 

The born rule must be applied, and it concerns the relative accessible histories. It is better to avoid the term “world” which is hard to define.

Accessible to who, or to what? AG 

Bruno Marchal

unread,
Jan 12, 2021, 7:47:19 AM1/12/21
to everyth...@googlegroups.com
On 6 Jan 2021, at 14:17, scerir via Everything List <everyth...@googlegroups.com> wrote:

Worlds, worlds. What are these worlds? When a pig observes a Young interferometer does this pig create worlds? Does this pig split worlds? Or not, because there is not full consciousness? And in Alpha Centauri,  where there are no pigs, no humans, no consciousness, no Young interferometers? No Franson interferometers either ...




It is better to avoid the word “world”. It is simpler to use histories, or even just “consistent relative memories”, which is what Everett use in QM (but works in a more fertile way starting from elementary arithmetic).

Then the MWI weight naturally (by tensor product and the wave equation, cf Gleason) the *accessible* (relative) histories on which we can bet when doing experiment. 

Worlds belong to the imagination of the relative universal numbers in elementary arithmetic. They are convenient fiction, unless mechanism is false, in which case we can *perhaps¨still make some sense of them. 

Bruno



--
Inviato da Libero Mail per Android

Mercoledì, 06 Gennaio 2021, 01:28PM +01:00 da Quentin Anciaux allc...@gmail.com:

Here a schema:
<Mail Attachment>

Bruno Marchal

unread,
Jan 12, 2021, 7:51:02 AM1/12/21
to everyth...@googlegroups.com
On 6 Jan 2021, at 14:17, Quentin Anciaux <allc...@gmail.com> wrote:

There are no this world... Every world is a world with a past. To simplify we will assume with start with one world at t, at t1, we have two worlds, at t2, 4 and so on... Each of these worlds are direct continuation of the unique world at time t, at t3, there is no *this worlds*, every alan in each worlds point to it saying this world... But there is no This world, it makes no sense.

It makes sense from the perspective of the one doing the experiment. He lost track of the “other worlds” because QM prevent the branch of the wave to interact (but not to interfere statistically).

Eventually the indexical “this” will ahem to be define using the machine’s theory of self-reference, where the formula 
B(Bp->p)->Bp constraints the accessible computations to obey some quantum logic, as confirmed by nature.

The word “world” should not be taken too much seriously, as it is misleading when we do metaphysics, theology or theories of everything…

Bruno




Quentin

Le mer. 6 janv. 2021 à 13:55, Alan Grayson <agrays...@gmail.com> a écrit :


On Wednesday, January 6, 2021 at 5:35:44 AM UTC-7 johnk...@gmail.com wrote:
On Wed, Jan 6, 2021 at 6:42 AM Alan Grayson <agrays...@gmail.com> wrote:

> There are things called laboratories, where physicists conduct experiments, some of which are quantum experiments with probabilistic outcomes.

If Many Worlds is correct then there are an astronomically large number, and possibly an infinitely large number, of worlds where physicists conduct experiments, some of which are quantum experiments with probabilistic outcomes.
 
The world in which such things exist, I call THIS world.

So there are an astronomically large number and possibly an infinitely large number of "THIS" worlds.

> Worlds postulated to exist based on the claim that any possible measurement, must be a realized measurement in another world, I call OTHER worlds.

Alan Grayson decides that tomorrow Alan Grayson will conduct an experiment to determine if an electron goes left or right. If Many Worlds is correct then the day after tomorrow one Alan Grayson will remember having seen the electron go left and one Alan Grayson will remember having seen the electron go right. Which Alan Grayson lives in "THIS" world.

> The core of my argument is that since the trial outcomes in quantum experiments are independent of one another, there's no reason to claim that each of the OTHER worlds accumulates ensembles, as an ensemble is created in THIS world.

That is just untrue. When one Alan Grayson has observed 1000 photons there is another Alan Grayson that agrees with 999 of the observations and disagrees only about #1000. All the 2^1000 Alan Graysons have made1000 observations, most Alan Graysons saw the electron go left about 500 times and go right about 500 times, but a few were quite different, one Alan Grayson out of  2^1000  saw the electron go left 1000 times in a row and one Alan Grayson out of 2^1000 saw the electron go right 1000 times in a row.

If you don't know what THIS world is, I can't help. AG 

 See my new list at  Extropolis

John K Clark

--
You received this message because you are subscribed to the Google Groups "Everything List" group.
To unsubscribe from this group and stop receiving emails from it, send an email to everything-li...@googlegroups.com.
To view this discussion on the web visit https://groups.google.com/d/msgid/everything-list/677fa7d5-1750-4347-8806-d500ce173430n%40googlegroups.com.

--
You received this message because you are subscribed to the Google Groups "Everything List" group.
To unsubscribe from this group and stop receiving emails from it, send an email to everything-li...@googlegroups.com.

Bruno Marchal

unread,
Jan 12, 2021, 7:53:30 AM1/12/21
to everyth...@googlegroups.com
On 6 Jan 2021, at 14:19, Quentin Anciaux <allc...@gmail.com> wrote:

I think there is no split, but continuous differentiation. So there is always an infinity of worlds. Or there is no world at all and only consciousness differentiation.

That’s better :)

Yes, in the arithmetical reality (aka the standard model of Arithmetic, the structure (N, 0, +, *) each universal number is the initial point of its many histories, where his consciousness start to diverge in infinitely many histories/consciousness stream.

Bruno




Quentin

Le mer. 6 janv. 2021 à 14:17, scerir via Everything List <everyth...@googlegroups.com> a écrit :

Worlds, worlds. What are these worlds? When a pig observes a Young interferometer does this pig create worlds? Does this pig split worlds? Or not, because there is not full consciousness? And in Alpha Centauri,  where there are no pigs, no humans, no consciousness, no Young interferometers? No Franson interferometers either ...

--
Inviato da Libero Mail per Android

Mercoledì, 06 Gennaio 2021, 01:28PM +01:00 da Quentin Anciaux allc...@gmail.com:

Here a schema:


Bruno Marchal

unread,
Jan 12, 2021, 7:56:28 AM1/12/21
to everyth...@googlegroups.com
On 6 Jan 2021, at 14:50, Terren Suydam <terren...@gmail.com> wrote:

This is how I see it as well. All possible worlds already exist in a platonic sense, and one's experience represents a single path traversed through the infinite multitude of possibilities. This connects nicely to the universal dovetailer idea.

And that point is even more convincing when we get that the existence of the (infinitely many) universal dovetailer is a theorem of elementary arithmetic (RA + Induction, i.e.. PA). We need to believe only that 2+2=4 and alike.

Bruno


Bruno Marchal

unread,
Jan 12, 2021, 8:01:18 AM1/12/21
to everyth...@googlegroups.com
Right. Like in the iterated self-duplication experiences. The duplicated person have no access to the memories of their doppelgangers. In Everett that explains the apparent wave reduction. In Mechanism, that seems to explain the origin of the Wave itself. The physical is a mathematical illusion, in the mind of the digital machine operating in virtue of the (sigma_1) arithmetical true relations.

Bruno



  John K Clark      See my new list at  Extropolis

--
You received this message because you are subscribed to the Google Groups "Everything List" group.
To unsubscribe from this group and stop receiving emails from it, send an email to everything-li...@googlegroups.com.

Bruno Marchal

unread,
Jan 12, 2021, 8:03:56 AM1/12/21
to everyth...@googlegroups.com
I see monothéisme as a popular version of monisme. It is the idea that there is ONE realm at the base of everything obersvable or conceivable. It is the rationalist idea that there is some explanation of everything, including consciousness and physical appearance, in a coherent fashion.

Bruno



John K Clark    See my new list at Extropolis

--
You received this message because you are subscribed to the Google Groups "Everything List" group.
To unsubscribe from this group and stop receiving emails from it, send an email to everything-li...@googlegroups.com.

Alan Grayson

unread,
Jan 12, 2021, 10:48:35 AM1/12/21
to Everything List
On Tuesday, January 12, 2021 at 5:41:07 AM UTC-7 Alan Grayson wrote:
On Tuesday, January 12, 2021 at 5:26:50 AM UTC-7 Bruno Marchal wrote:

On 3 Jan 2021, at 03:43, Alan Grayson <agrays...@gmail.com> wrote:



On Saturday, January 2, 2021 at 2:17:12 AM UTC-7 johnk...@gmail.com wrote:
On Fri, Jan 1, 2021 at 5:35 PM 'Brent Meeker' via Everything List <everyth...@googlegroups.com> wrote:

>> Assuming that Many Worlds is true and the multiverse is completely determined by Schrodinger's equation and there are therefore an astronomically large number (perhaps an infinite number) of Bruce Kelletts with microscopic or submicroscopic differences between them, and those Bruce Kelletts were observing a stream of photons polarized at angle X hit a polarizing filter set to angle X+Y; would any one of those Bruce Kelletts be able to predict with certainty that Bruce Kellett would or would not observe the photon pass through that filter? No. Would Bruce Kellett have to resort to probability? Yes. How would Bruce Kellett calculate the probability? If Bruce Kellett wanted to avoid logical self contradictions there is only one method Bruce Kellett could use, the Born Rule.

> I don't think that's quite true.  Suppose for example BK decided to predict that the polarization with the highest value of |psi|^2 is the one that would pass thru. He wouldn't run into any logical contradiction because he's not interpreting it as probability,

If the BKs are Interpreting that as a certainty and not a probability then the BKs wouldn't run into a logical contradiction but they would run into an empirical one because that wouldn't match experimental observation. It's entirely possible that a BK's prediction would fail and that the high |psi|^2 photon would NOT make it through (unless the value happened to be exactly 1), and even if the prediction turned out to be correct scientific experiments must be repeatable and when the BKs conduct it over and over again all the BKs will soon find out that the predictions tend to be correct |psi|^2 of the time.

 > he wouldn't run into an empirical contradiction unless he assumed the actual process was producing a probability distribution and so he needed to predict a distribution and not just a value. 

But the BKs didn't assume it was a probability distribution, they discovered it was. If the BKs assumed the |psi|^2 value was just a number and not a probability and had no physical significance then the BKs would soon discover that the assumption was wrong
  
> Once you know that you need a probability distribution from the wave function...then Born's rule is the only choice. 

Yes.
 
> But it's the step from the wave-function and "everything happens" to a probability distribution where MWI leaves a gap.

I don't see the gap. If Many Worlds was true then what would the Brent Meekers interpret |psi|^2 to mean? If it's just a number and means nothing then solving Schrodinger's equation would be a waste of time because that equation would also mean nothing, it should be ignored; but then we wouldn't have transistors or lasers or about 6.02*10^23 other things in modern life. 

The gap Brent refers to has nothing to do with Schrodinger's equation, as I previously explained. Every trial in an experiment can be interpreted as a separate horse race, creating its own set of worlds where each possible occurrence is allegedly measured. But on subsequent trials, the MWI gives no guarantee that the same set of worlds is created. IOW, without another postulate appended to the MWI, each world is associated with exactly ONE measurement. No ensembles in these worlds; hence, the necessary condition for a probability doesn't exist. AG. 

The born rule must be applied, and it concerns the relative accessible histories. It is better to avoid the term “world” which is hard to define.

Accessible to who, or to what? AG 

IMO, it's impossible to avoid using the term "world". After all, the MWI depends on the claim that everything that CAN happen, MUST happen. IOW, every possible measurement MUST be measured, somewhere, somehow. Can you measure an event without an observer and measuring device? The "observer" doesn't have to be human. It could be an instrument. But whatever it is, it surely DOES imply a "world" of some sort, partially or fully.  And once you admit that other "worlds" necessarily come into existence given the core assumption on which the MWI depends, the entire structure of the interpretation falls away, into absurdity; e.g., where is the energy to create these worlds? AG

Pierz

unread,
Jan 13, 2021, 12:19:59 AM1/13/21
to Everything List


On Monday, January 4, 2021 at 12:09:06 PM UTC+11 agrays...@gmail.com wrote:
On Sunday, January 3, 2021 at 3:56:51 PM UTC-7 johnk...@gmail.com wrote:
On Sun, Jan 3, 2021 at 5:21 PM Alan Grayson <agrays...@gmail.com> wrote:

> The MWI doesn't guarantee that these subsequent measurements, for subsequent horse races say, are occurring in the SAME OTHER worlds as trials progress, to get ensembles in those OTHER worlds. 
 
I don't know what you mean by "SAME OTHER worlds", the same as what? In one world Alan Grayson remembers having seen the electron go left, in another world Alan Grayson remembers having seen the electron go right, other than that the two worlds are absolutely identical, so which one was the "SAME OTHER world"?

> You seem to avoid the fact that no where does the MWI guarantee [...]

Quantum mechanics is not in the guarantee business, it deals with probability.  

> I don't think you understand my point, which isn't complicated. 

Yes, your point is very simple indeed, but the word simple can have 2 meanings, one of them is complementary and the other not so much.  

In first trial, the MWI postulates other worlds comes into existence. Same other worlds in second trial? Same other worlds in third trial, etc? Where does the MWI assert these other worlds are the SAME other worlds? Unless it does, you only have ONE measurement in each of these worlds. No probability exists in these other worlds since no ensemble of measurements exist in these other world. AG
 
You grossly misunderstand MWI. There are no "same other" worlds. The worlds that arise at each trial are different in precisely one way and one way only: the eigenvalue recorded for the experiment. The different eigenvalues will then give rise to a "wave of differentiations" as the consequences of that singular difference ramifies, causing the different worlds generated by the original experimental difference to multiply. "World" really means a unique configuration of the universal wave function, so two worlds at different trials can't possibly be the "same world", and yes, there is only one measurement in each. That is precisely the stipulation of MWI. If we have a quantum experiment with two eigenvalues 1 and 0, and each is equally likely per the Born rule, then the MWI interpretation is that - effectively - two worlds are created. You, the experimenter, end up in both, each version knowing nothing about the other. So, in the "objective world" (the view from outside the whole wave function as it were), no probability is involved. But if you repeat this experiment many times, each version of you will record an apparently random sequence of 1s and 0s. Your best prediction of what happens in the next experiment is that it's a 50/50 toss up between 1 and 0. Objectively there's no randomness, subjectively it appears that way.

Alan Grayson

unread,
Jan 13, 2021, 1:50:29 AM1/13/21
to Everything List
On Tuesday, January 12, 2021 at 10:19:59 PM UTC-7 Pierz wrote:


On Monday, January 4, 2021 at 12:09:06 PM UTC+11 agrays...@gmail.com wrote:
On Sunday, January 3, 2021 at 3:56:51 PM UTC-7 johnk...@gmail.com wrote:
On Sun, Jan 3, 2021 at 5:21 PM Alan Grayson <agrays...@gmail.com> wrote:

> The MWI doesn't guarantee that these subsequent measurements, for subsequent horse races say, are occurring in the SAME OTHER worlds as trials progress, to get ensembles in those OTHER worlds. 
 
I don't know what you mean by "SAME OTHER worlds", the same as what? In one world Alan Grayson remembers having seen the electron go left, in another world Alan Grayson remembers having seen the electron go right, other than that the two worlds are absolutely identical, so which one was the "SAME OTHER world"?

> You seem to avoid the fact that no where does the MWI guarantee [...]

Quantum mechanics is not in the guarantee business, it deals with probability.  

> I don't think you understand my point, which isn't complicated. 

Yes, your point is very simple indeed, but the word simple can have 2 meanings, one of them is complementary and the other not so much.  

In first trial, the MWI postulates other worlds comes into existence. Same other worlds in second trial? Same other worlds in third trial, etc? Where does the MWI assert these other worlds are the SAME other worlds? Unless it does, you only have ONE measurement in each of these worlds. No probability exists in these other worlds since no ensemble of measurements exist in these other world. AG
 
You grossly misunderstand MWI. There are no "same other" worlds. The worlds that arise at each trial are different in precisely one way and one way only: the eigenvalue recorded for the experiment. The different eigenvalues will then give rise to a "wave of differentiations" as the consequences of that singular difference ramifies, causing the different worlds generated by the original experimental difference to multiply. "World" really means a unique configuration of the universal wave function, so two worlds at different trials can't possibly be the "same world", and yes, there is only one measurement in each.

This is what I have been saying all along! AG
 
That is precisely the stipulation of MWI. If we have a quantum experiment with two eigenvalues 1 and 0, and each is equally likely per the Born rule, then the MWI interpretation is that - effectively - two worlds are created. You, the experimenter, end up in both, each version knowing nothing about the other.

Again, what I have been saying all along! AG
 
So, in the "objective world" (the view from outside the whole wave function as it were), no probability is involved. But if you repeat this experiment many times, each version of you will record an apparently random sequence of 1s and 0s. Your best prediction of what happens in the next experiment is that it's a 50/50 toss up between 1 and 0. Objectively there's no randomness, subjectively it appears that way.
 
Here's where you go astray. AG 

Pierz

unread,
Jan 13, 2021, 6:33:20 PM1/13/21
to Everything List
On Wednesday, January 13, 2021 at 5:50:29 PM UTC+11 agrays...@gmail.com wrote:
On Tuesday, January 12, 2021 at 10:19:59 PM UTC-7 Pierz wrote:


On Monday, January 4, 2021 at 12:09:06 PM UTC+11 agrays...@gmail.com wrote:
On Sunday, January 3, 2021 at 3:56:51 PM UTC-7 johnk...@gmail.com wrote:
On Sun, Jan 3, 2021 at 5:21 PM Alan Grayson <agrays...@gmail.com> wrote:

> The MWI doesn't guarantee that these subsequent measurements, for subsequent horse races say, are occurring in the SAME OTHER worlds as trials progress, to get ensembles in those OTHER worlds. 
 
I don't know what you mean by "SAME OTHER worlds", the same as what? In one world Alan Grayson remembers having seen the electron go left, in another world Alan Grayson remembers having seen the electron go right, other than that the two worlds are absolutely identical, so which one was the "SAME OTHER world"?

> You seem to avoid the fact that no where does the MWI guarantee [...]

Quantum mechanics is not in the guarantee business, it deals with probability.  

> I don't think you understand my point, which isn't complicated. 

Yes, your point is very simple indeed, but the word simple can have 2 meanings, one of them is complementary and the other not so much.  

In first trial, the MWI postulates other worlds comes into existence. Same other worlds in second trial? Same other worlds in third trial, etc? Where does the MWI assert these other worlds are the SAME other worlds? Unless it does, you only have ONE measurement in each of these worlds. No probability exists in these other worlds since no ensemble of measurements exist in these other world. AG
 
You grossly misunderstand MWI. There are no "same other" worlds. The worlds that arise at each trial are different in precisely one way and one way only: the eigenvalue recorded for the experiment. The different eigenvalues will then give rise to a "wave of differentiations" as the consequences of that singular difference ramifies, causing the different worlds generated by the original experimental difference to multiply. "World" really means a unique configuration of the universal wave function, so two worlds at different trials can't possibly be the "same world", and yes, there is only one measurement in each.

This is what I have been saying all along! AG
No it isn't. I agree you have been saying there is only one measurement outcome in each world. However this business about "same other worlds" betrays your lack of comprehension. It's not that MWI "doesn't guarantee" that the the worlds at each trial are the same world. It's that the whole notion of "same other worlds" means nothing in this context and has no bearing on anything. A bit like arguing when we add 1 and 1 twice whether we are guaranteed that the ones we add each time are the "SAME ones" at each addition. If mathematics can't guarantee that then how can we be sure the answer is the same? Basically the only answer to that is "WTF?"
 
That is precisely the stipulation of MWI. If we have a quantum experiment with two eigenvalues 1 and 0, and each is equally likely per the Born rule, then the MWI interpretation is that - effectively - two worlds are created. You, the experimenter, end up in both, each version knowing nothing about the other.

Again, what I have been saying all along! AG
If you get that, then the next bit follows. 
 
So, in the "objective world" (the view from outside the whole wave function as it were), no probability is involved. But if you repeat this experiment many times, each version of you will record an apparently random sequence of 1s and 0s. Your best prediction of what happens in the next experiment is that it's a 50/50 toss up between 1 and 0. Objectively there's no randomness, subjectively it appears that way.
 
Here's where you go astray. AG 
 
So you say! Without justifying yourself in any way. You seem to be saying that probability can't describe QM experiments because in each world there is only one outcome and therefore no "ensemble" of outcomes from which a probability can be derived. That is totally wrong-headed. There are two "ensembles": the ensemble of different multiverse branches at each experiment, and the ensemble of each experimenter's prior measurements, and those are enough to derive the appearance of randomness and to justify a probabilistic description despite the objective lack of randomness. If you agree with "what you have been saying all along", then you must agree that every experimenter in every world in an MWI multiverse will see a record of an apparently random sequence of 1s and 0s in the described experiment. Right? And if not why not? 

Alan Grayson

unread,
Jan 13, 2021, 9:23:11 PM1/13/21
to Everything List
IMO, since the trials are independent, the other observers are disjoint from each other and each records only one measurement. So the only observer who sees an ensemble is the observer in THIS world. To get an ensemble of outcomes in those other worlds, and hence a probability, you need to appeal to a non-existent observer, also called the Bird's Eye observer. AG 

Pierz

unread,
Jan 13, 2021, 10:29:16 PM1/13/21
to Everything List
Respectfully, you did not answer my question. Do you agree or not that every experimenter in every branch of the multiverse who records a series of experiments as described in my scenario will record a seemingly random string of 1s and 0s? If you do, that's really all that's required. Abstract debates about "ensembles required to get a probability" are moot. If the world is as described by MWI, the appearance of probability is an outcome, and probability is the best possible description of how quantum experiments turn out from any real observer's POV (as opposed to the Bird's Eye observer). If you disagree that experimenters will get a seemingly random string of 1s and 0s, then you'll need to explain why you think that.

Alan Grayson

unread,
Jan 13, 2021, 10:42:43 PM1/13/21
to Everything List
I did answer your question. Since the trials are independent, a NEW OTHER WORLD observer is created on each THIS WORLD trial. So the other observers see ONE outcome each. No reason to assume otherwise. You need another postulate for this to work. AG 

Quentin Anciaux

unread,
Jan 14, 2021, 1:21:45 AM1/14/21
to everyth...@googlegroups.com
--

So your postulate is that at each experiment, the other AG has no past memory and is as conscious as a rock right ? Because if not, those "others" AG have the same past memory as "this' AG and have an ensemble of results consisting of results of the past trials in their memory and the latest trial.

Quentin
You received this message because you are subscribed to the Google Groups "Everything List" group.
To unsubscribe from this group and stop receiving emails from it, send an email to everything-li...@googlegroups.com.

Quentin Anciaux

unread,
Jan 14, 2021, 1:23:27 AM1/14/21
to everyth...@googlegroups.com
A little correction here before you go again in the WTF argument. They have the same past memory with this AG up until they spiltted/differentiated from him.

Pierz

unread,
Jan 14, 2021, 4:26:42 AM1/14/21
to Everything List
You're talking like a politician. Does each observer in each world who repeats said experiment record a seemingly random series of 1s and 0s or not? Yes/no. It's not hard. Come on, you can do it now...

scerir

unread,
Jan 14, 2021, 6:52:16 AM1/14/21
to everyth...@googlegroups.com

IMO the main quantum postulate is the following. 'Real experiments have results. Unperformed experiments have none.' (But we can create different postulates, and different theories. Only future experiments will tell ...)

--

Alan Grayson

unread,
Jan 14, 2021, 7:07:59 AM1/14/21
to Everything List
The answer is NO. In the spin experiment we're discussing, AG in this world measures an apparently random sequence of 0's and 1's. On each trial an imaginary other AG measures the complement of what AG in this world measured. Now since the trials are independent, different imaginary AGs always measure complements, but none measure more than ONE RESULT. As I previously indicated, these other AGs are disjoint from each other. The only way to remedy this situation is to add another postulate to your MWI. AG

Alan Grayson

unread,
Jan 14, 2021, 7:13:18 AM1/14/21
to Everything List
On Thursday, January 14, 2021 at 4:52:16 AM UTC-7 sce...@libero.it wrote:

IMO the main quantum postulate is the following. 'Real experiments have results. Unperformed experiments have none.' (But we can create different postulates, and different theories. Only future experiments will tell ...)


I think we're in agreement. IMO, those other AGs aren't really measuring anything. They're imaginary constructs, "forced" to "measure" the complement of whatever this world AG measures. AG

Pierz

unread,
Jan 14, 2021, 5:15:47 PM1/14/21
to Everything List
That's all we need. Remember, we are talking about if MWI were true here. This is a thought experiment in which we are imagining how things would go if the MWI picture were correct. So if you're admitting that, in such a world, "AG in this world measures an apparently random sequence of 0's and 1's", then you have all the preconditions for making probabilistic predictions from it. 
On each trial an imaginary other AG measures the complement of what AG in this world measured.
 
They are not imaginary in MWI. I agree with this statement once you remove the word "imaginary", which we can do ex-hypothesi. I'm not asking you to believe in them here, only to advance an argument based of "if" MWI were true.
Now since the trials are independent, different imaginary AGs always measure complements, but none measure more than ONE RESULT.
 
Again, discarding the word "imaginary", I agree. Of course they don't measure more than one outcome at the same trial. 
As I previously indicated, these other AGs are disjoint from each other.
 
What do you mean by "disjoint" exactly? Mathematically "disjoint" means "having no elements in common". In the case of AG's who have measured different results, initially their worlds have only this difference between them, so in that sense they are not disjoint. If you mean they cannot interact with one another, and inhabit diverging realities, then that is only the case discounting interference, which we cannot do, because without interference effects we dot have quantum mechanics. This word disjoint seems to be central to your objection, but you need to define precisely what is meant by it or we cannot assess the validity of your claims.

The only way to remedy this situation is to add another postulate to your MWI. AG
 
No idea what we need to remedy. I'll ask my question again, adjusting it slightly. Does AG record a seemingly a random string of 1s and 0s in this experiment if MWI describes reality?

 

Philip Benjamin

unread,
Jan 14, 2021, 6:01:41 PM1/14/21
to everyth...@googlegroups.com, general...@googlegroups.com

[Philip Benjamin]

        Be it Relativity Theory (actually Relationality) where Social Sciences ignore the speed of light in vacuo as a CONSTANT (ABSOLUTE) or Quantum Mechanics where mystics disregard the AS IF Logic but accept the BOTH & Fallacy of de Broglie’s wave-like, not Bohr’s wavy, particle, or be it Evolution qua Trans-speciation where the acade-media substitute the un-evidential inter-species Trans-speciation for the evidential intra-species Adaptation or Variation (i.e. Natural Selection). The unproven dogmas of the acade-media have both direct and indirect deleterious effects on social norms in general and current events in particular. They ignore the inevitable questions of aseity, causality, infinite regress, origin, morality, meaning, teleology etc. All dictatorial systems where pagans with reptilian/kundalini, un-awakened consciousnesses dominate, have some or all of these beliefs as their foundations (e.g. pagan Socialist Hitler, pagan Marxist Stalin, pagan Fascist Mussolini and all similar pagan Progressives). Pagan = Pan-Gaian = earthlings for whom earth is all that matters).

                                                                     Western Civilization is an Augustinian Trust

                                                                      Its stealing beneficiary is WAMP-the-Ingrate

                                          WAMP = Western Acade-Media Pagan(ism), a parody of the erstwhile WASP

        Paganism, be civilized and scholarly, is un-awakened kundalini consciousness. Non-paganism is awakened, non-reptilian consciousness. Pagan hatred for non-pagans is atavistic, unilateral and universal—pagan Cain vs non-pagan sibling Abel!! The Puritan/Reformation idea of inalienable personal rights for “Life, Liberty and Private Property, as divinely ordained birthrights not bestowed by law, custom, or belief, and which cannot be taken or given away, or transferred to another person, is totally foreign to the WAMP sense of individual freedoms (licentiousness). https://www.bizpacreview.com/2021/01/11/dershowitz-destroys-democrats-impeachment-hopes-1015126/. Modern civilized pagan politicians may impeach all past presidents from George Washington to 46-th.   

       Argumentum verecundiam may use respect for the brilliance and authority of an American physicist Hugh Everett (1957) in order to fortify one’s argument and offer an impression or “hunch” of proof for propositions of questionable validity and reliance such as The Multiverse speculation that “there are many worlds which exist in parallel at the same space and time as our own”. Here, the Schrodinger equation never collapses, but all probabilities of a quantum superstition are objectively real,  unlike the subjective CopenPagan Interpretation (a malaprop for Copenhagen!).  This makes it possible to remove randomness and action at a distance from quantum theory, but that does not make it true. Quantum theory interpretations, be it the subjective CopenPagan or objective Many Worlds, are based on imaginary conceptualization of particle-wave duality. De Broglie wave-length is based only on apparent wave-likeness not actual waviness of particles. An AS IF Logic, not Both & Fallacy, is all that is needed here.

       Photons also are corpuscular, probably with mas at an indefinite decimal place, behaving AS IF in wave forms. Corpuscular photon and particulate matter are very differently treated.  In 1905 Einstein managed to write E^2=P^2c^2+m^2c^4,and he figured out that light is both a particle and a wave and that the energy of a photon isn’t governed by its mass or its velocity (like matter), instead it is governed entirely by its frequency, f: E=hf, where h is Planck’s constant 6.626070150 × 10-34 kgm2/s which nonetheless has kg (mass) in its unit. Photons may have mass.  https://www.askamathematician.com/2010/09/q-how-can-photons-have-energy-and-momentum-but-no-mass/https://science.howstuffworks.com/math-concepts/kilogram-is-dead-meet-kilogram-20.htm. Imaginary complex numbers are used in practically every field of physics and mathematics, in electrical engineering and control system design. That does not make square root of minus 1 real, though all numbers by themselves are abstractions.   

       A set of data may be of objective science, but interpretation of that set is a subjective stuff. World views of scientists will be ALWAYS a determining factor in their interpretations of observations or data. Civilized pagan atheists or humanists with an un-awakened reptilian/kundalini consciousness, such as Hugh Everett, will not have the same interpretation as civilized theists like Galileo or Copernicus or Newton or Faraday or Joule, each with an Augustinian awakened consciousness (https://www.midwestaugustinians.org/conversion-of-st-augustine). Augustine was once a civilized erudite pagan of Greco Roman roots, who baptized Platonism into Scriptural norms,  pulled the West out of Greco-Roman paganism and is considered the chief architect of Western Civilization based on Transformation or Rebirth of the “self” (inner man) solely (100%) by the extrinsic source of Power of Adonai (plural) YHWH (singular) Elohim (uni-plural) of the Patriarchs, Prophets and the Apostles. Augustine in fact built on the foundation of the Athenian Mars Hill discourse by Rabbi Saul of Tarsus (Acts Ch. 17) where the Greek Unknown god was identified as the Risen Messiah. The American Puritan concept of a Non-sovereign Federation of the Sovereign States is clearly not the result of atheistic-humanist paganism, pagan politics, New Age occultism, Taoism, Talmudism, TM, Jungian Sorceries, the Koran or the Puranas etc. In reality, America is the product of “Two Great Awakenings”, both historical and historic. Only a Third Great Awakening of individual consciousness on a massive scale is the remedy for the present malady. Pagans with un-awakened consciousness are now predominant everywhere: pulpits and pews, politics and parties, bureaucracies and businesses, academia and media, military and militia.                                                                       

       For science today, the only candidate available for the “inner man” or “self” of Augustinian philosophy (as he understood from the Scriptures) is the bio dark-matter body with its bio dark-matter chemistries formed at the moment of conception along with the bio light-matter body with its chemistries. The subatomic constituents a dark-atom will have negligible masses with respect to electron, but the same mass ratios as those in light-atom. The dark and light twins are in resonance, offering a basis for self-consciousness. Resonance is rudimentary recognition. A non-electric, non-entropic, durable and precocial bio dark-matter twin, co-created at the moment of conception, can survive the altricial light-matter twin. When decoupled at death the bio dark-matter body will be relatively at a negative energy state by -E = mC^2 where m is the dead body mass. That will be the magnitude of the threshold external energy needed to raise the bio dark matter body to any functional state (Physical resurrection for example).

 

Evidentialist

Philip Benjamin                           

                                                            CC. Prof. Robert B. Reich, Berkeley

Freedom From Religion Foundationhttps://ffrf.org/news/day/dayitems/item/14896-hugh-everett-iii  “Everett was a “lifelong atheist,” according to The Many Worlds of Hugh Everett III (2010) by Peter Byrne. During his time at the Catholic university, Everett “drove devout Jesuits to distraction with scientific questioning” and even caused one of his professors to lose his faith after presenting a logical proof against the existence of God (quoted in The Many Worlds of Hugh Everett III).”

https://ffrf.org/faq/item/14999-what-is-the-foundations-purpose  “The history of Western civilization shows us that most social and moral progress has been brought about by persons free from religion. In modern times, the first to speak out for prison reform, for humane treatment of the mentally ill, for abolition of capital punishment, for women’s right to vote, for death with dignity for the terminally ill, and for the right to choose contraception, sterilization and abortion have been freethinkers, just as they were the first to call for an end to slavery”. [Note: This is blatantly wrong. Take for example the universal bane of slavery. Thanks to the life-long efforts of the Puritan MP William Wilberforce, for the first time in human history Slave Trade was abolished by law in the British Empire—the vastest, greatest, most powerful, boblest (not perfect) empire ever on planet earth. It was followed six decades later by the American Emancipation. Modern civilized pagan politicians may impeach all past presidents from George Washington to the 46- th.]

                                        

Any ONE? “This is why you seem to "argue" like a Trumper…” If Many Worlds is correct then an astronomical number of Alan Graysons remember witnessing a physicist measure a quantum event, but the Alan Graysons will not all have the same memories”.

https://www.huffpost.com/entry/trump-fascism-future-ruth-ben-ghiat_n_5ff8c215c5b6c77d85e6a3d0 ; https://www.msn.com/en-us/news/politics/hes-going-to-be-more-unleashed-and-unhinged-than-ever/  Ruth Ben-Ghiat is a history professor at New York University and author of the book “Strongmen: Mussolini to the Present.”  She talked to HuffPost about how she thinks Trump might act once he’s out of office and how America could be headed for even more violence and political unrest.He's Going To Be More Unleashed And Unhinged Than Ever…. Ruth Ben-Ghiat is a history professor at New York University and author of the book “Strongmen: Mussolini to the Present.”  She talked to HuffPost about how she thinks Trump might act once he’s out of office and how America could be headed for

even more violence and political unrest…. Christopher Mathias  01/09/2021 08:00 am ET.  You’ve described this kind of leader-follower relationship like a fascistic relationship. Typically how is that spell broken? How do people get out of that”?

https://sqpn.com/2021/01/what-is-the-great-reset-davos-and-the-world-economic-forum/    MYS136: The World Economic Forum has proposed to use the Covid-19 pandemic as an opportunity to have a “Great Reset” to fundamentally transform the way the world works.

https://republicanpress.org/world-economic-forum-calls-for-great-reset-of-capitalism/ (RepublicanPress.org) – The world is on the brink of an economic collapse due to China’s mishandling of the COVID-19 pandemic. As if that wasn’t bad enough, the World Economic Forum (WEF) appears to be trying to reset capitalism to socialism in response to the crisis. The potential fallout to the world’s economies could be immeasurable, if this turns out to be true. The WEF plan is called The Great Reset Initiative, and a quick review of its official website is enough to alarm anyone concerned about the continuation of American economic values….. The site hones in on the geopolitical, social, and political disruptions occurring in the wake of the COVID-19 outbreak. Incredibly, it appears to interpret the crisis as “a unique window of opportunity” to shape the future national economies. It seems to draw a connection between socialistic values and, somehow, links them to “the dignity of every human being.”

https://businessmirror.com.ph/2021/01/07/world-leaders-urge-us-china-reset-at-bloomberg-new-economy-forum/ “The first day of the four-day Bloomberg New Economy Forum concluded with business and political leaders taking on issues from the economic fallout of the coronavirus pandemic to the future of global trade and climate change…. The New Economy Forum is organized by Bloomberg Media Group, a division of Bloomberg LP, the parent company of Bloomberg News.”.

   https://slate.com/news-and-politics/2021/01/biden-pelosi-schumer-john-lewis-save-democracy.html . Slate. Richard L. Hasen. 1-11-2021. The Only Way to Save American Democracy Now.    “Right now, it takes 60 votes to get most things done in the Senate, a structure that helps perpetuate minority rule. In the Senate, small Republican states like Wyoming, with fewer than 600,000 people, can join together to thwart the voting rights of states like California, with nearly 40 million people. (In the last Congress, for instance, senators representing 13 million fewer voters commanded a 53–47 Republican Senate majority.) A filibuster exception for voting rights legislation helps to negate that anti-majoritarian advantage in the Senate….. We should not allow state legislatures or canvassing boards to easily overturn election results, and the threshold for debating objections to Electoral College votes in Congress needs to be raised substantially”.  https://tribunecontentagency.com/article/robert-b-reich-trump-attempted-a-coup-and-must-be-removed/  January 11, 2021 Robert B. Reich: “Trump attempted a coup and must be removed…. Amendment right to try to overthrow the U.S. government. Trump’s accomplices on Capitol Hill, most notably Texas Sen. Ted Cruz and Missouri Sen. Josh Hawley, should be forced to resign… Donald Trump. son Donald Trump Jr. and personal attorney Rudy Giuliani should be areested and tried for sedition…”

~~~~~~~~~~~~~~~~~~~~~~~~~~~~~~~~~~~~~~~~~~~~~~~~~~~~~~~~~~~~~~~~~~~~~~~~~~~~~~~~~~~~~~~~~~~~~~~~~~~~~~~~~~~~~~~~~~~~~~~~~~~~~~~~~~~~~~~~~~~~~~~~~~~~~~~~~~

 

From: everyth...@googlegroups.com <everyth...@googlegroups.com> On Behalf Of John Clark
Sent: Friday, January 8, 2021 8:59 AM To: everyth...@googlegroups.com
Subject: Re: Born's rule from almost nothing

On Thu, Jan 7, 2021 at 11:15 AM Philip Benjamin <medin...@hotmail.com> wrote:

 

[Philip Benjamin] > The whole idea of multiverse is contrary to all laws of logic

 

[John K ClarkMy hunch is that Many Worlds Is correct but I could be wrong, but right or wrong it doesn't violate any law of logic, it's odd but not paradoxical and we already know from experiment that whatever the true interpretation of Quantum Mechanics turns out to be it's going to be odd, very very odd.

 

[Philip Benjamin] >  and the fundamental physical law of conservation of mass and energy.

 

[John K Clark ] Unlike the Second Law Of Thermodynamics which we know from pure logic must be true a violation in the law of conservation of mass and energy would not violate any laws of logic. And even before Quantum Mechanics came along we knew from General Relativity that the law of conservation of mass-energy only holds true locally not globally. The universe is expanding so photons of light get stretched to a longer wavelength and photons with a longer wavelength have less energy, so where did that energy go? It didn't go anywhere, it just disappeared. And now we know that the universe is accelerating, so where did the energy for that acceleration come from? It came from empty space. It's still true that if a spherical region of space of fixed size is in thermal equilibrium then the amount of energy entering that sphere will equal the amount of energy leaving it, it's just that now more space has been created outside that sphere and thus there is more energy.  

 

[Philip Benjamin] > Augustine’s consciousness was instantly transformed form paganism to non-paganism

[John K Clark] Personally I don't think the change from polytheism to monotheism was much of an advance, both ideas are silly but polytheism is more fun, it makes for better comic books. 

 

John K Clark   

.

Philip Benjamin

unread,
Jan 14, 2021, 6:10:46 PM1/14/21
to everyth...@googlegroups.com

general...@googlegroups.com Subject: [Consciousness-Online] RE: Born's rule from almost nothing

 

[Philip Benjamin]

        Be it Relativity Theory (actually Relationality) where Social Sciences ignore the speed of light in vacuo as a CONSTANT (ABSOLUTE) or Quantum Mechanics where mystics disregard the AS IF Logic but accept the BOTH & Fallacy of de Broglie’s wave-like, not Bohr’s wavy, particle, or be it Evolution qua Trans-speciation where the acade-media substitute the un-evidential inter-species Trans-speciation for the evidential intra-species Adaptation or Variation (i.e. Natural Selection), the unproven dogmas of the acade-media have both direct and indirect deleterious effects on social norms in general and current events in particular. They ignore the inevitable questions of aseity, causality, infinite regress, origin, morality, meaning, teleology etc. All dictatorial systems where pagans with reptilian/kundalini, un-awakened consciousnesses dominate, have some or all of these beliefs as their foundations (e.g. pagan Socialist Hitler, pagan Marxist Stalin, pagan Fascist Mussolini and all similar pagan Progressives). Pagan = Pan-Gaian = earthlings for whom earth is all that matters).

Alan Grayson

unread,
Jan 15, 2021, 12:01:38 AM1/15/21
to Everything List
I meant, of course, that each other AG measures ONE value when created for some trial, but that's all this observer EVER observes because, unless you can offer an argument, he's disjoint, that is NOT related or connected any other "other AG". AG 

As I previously indicated, these other AGs are disjoint from each other.
 
What do you mean by "disjoint" exactly? Mathematically "disjoint" means "having no elements in common". In the case of AG's who have measured different results, initially their worlds have only this difference between them, so in that sense they are not disjoint. If you mean they cannot interact with one another, and inhabit diverging realities, then that is only the case discounting interference, which we cannot do, because without interference effects we dot have quantum mechanics. This word disjoint seems to be central to your objection, but you need to define precisely what is meant by it or we cannot assess the validity of your claims.

The only way to remedy this situation is to add another postulate to your MWI. AG
 
No idea what we need to remedy. I'll ask my question again, adjusting it slightly. Does AG record a seemingly a random string of 1s and 0s in this experiment if MWI describes reality?

You seem to be assuming the other worlds created according to the MWI interact with other due to interference. Since these other worlds are never observed, I call them "imaginary"; and more important, no observations of interacting other worlds have ever been made, within QM or without QM. So the MWI is a huge stretch, at best. AG 

 

Pierz Newton-John

unread,
Jan 15, 2021, 1:36:39 AM1/15/21
to everyth...@googlegroups.com
Of course? I’m starting to think your picture of MWI is even more misconceived than I imagined. Look, the MWI multiverse has the structure of a tree, where each branch event is a different possible eigenvalue of some observable. But - again ex hypothesi - all branches of the tree are ontologically equal. They are all as real as one another. Each AG’s past at a particular trial is the same as all the other AGs at that trial, though each AG’s future diverges into further trees of different observed realities. The equal status of all branches means that your idea that AG observers on other branches are in some way different from the AG branch you happen to be on in that they only ever have one observation is just totally misconceived. To be blunt, you don’t get it.

As I previously indicated, these other AGs are disjoint from each other.
 
What do you mean by "disjoint" exactly? Mathematically "disjoint" means "having no elements in common". In the case of AG's who have measured different results, initially their worlds have only this difference between them, so in that sense they are not disjoint. If you mean they cannot interact with one another, and inhabit diverging realities, then that is only the case discounting interference, which we cannot do, because without interference effects we dot have quantum mechanics. This word disjoint seems to be central to your objection, but you need to define precisely what is meant by it or we cannot assess the validity of your claims.

The only way to remedy this situation is to add another postulate to your MWI. AG
 
No idea what we need to remedy. I'll ask my question again, adjusting it slightly. Does AG record a seemingly a random string of 1s and 0s in this experiment if MWI describes reality?

You seem to be assuming the other worlds created according to the MWI interact with other due to interference. Since these other worlds are never observed, I call them "imaginary"; and more important, no observations of interacting other worlds have ever been made, within QM or without QM. So the MWI is a huge stretch, at best. AG

Im not assuming it. It’s part of QM that wave functions interfere with themselves. In MWI that translates to different “worlds” interfering. That interference is extremely limited since it only occurs to the extent that two branches can become identical again after having diverged. Nonetheless it is the basis for proposed experimental proofs of MWI. One argument goes that if we can make a quantum computer with a sufficiently large number of qubits, we can prove the existence of other worlds because the other worlds are the only place we can get all that information from. So, no, this is not my assumption, it’s intrinsic to MWI. Again, if you don’t get that, you don’t get MWI. You can’t expect your arguments to be taken seriously if you don’t understand basics like this.

 

--
You received this message because you are subscribed to a topic in the Google Groups "Everything List" group.
To unsubscribe from this topic, visit https://groups.google.com/d/topic/everything-list/xsl8cSDT4M8/unsubscribe.
To unsubscribe from this group and all its topics, send an email to everything-li...@googlegroups.com.
To view this discussion on the web visit https://groups.google.com/d/msgid/everything-list/9028f9b4-f111-4366-bb84-f4024d15202do%40googlegroups.com.

Quentin Anciaux

unread,
Jan 15, 2021, 2:48:14 AM1/15/21
to everyth...@googlegroups.com
Obviously I'm blocked by AG for whatever reason, because that's what I've been telling him since and got no answer... I even bother to take the time to make a schematics (well a ugly one but still) :D

You received this message because you are subscribed to the Google Groups "Everything List" group.
To unsubscribe from this group and stop receiving emails from it, send an email to everything-li...@googlegroups.com.
To view this discussion on the web visit https://groups.google.com/d/msgid/everything-list/CAArMS00gPngPUgNvQLBvjOAHN%2B0TVVkBC%2BtYSJSah_5PNSx9qQ%40mail.gmail.com.

Alan Grayson

unread,
Jan 15, 2021, 3:25:15 AM1/15/21
to Everything List
No. You're not blocked. I'll try to look at it and give a response. AG

Alan Grayson

unread,
Jan 15, 2021, 1:56:27 PM1/15/21
to Everything List
I'm really dumb. Ask Bruce for confirmation. I agree; in a horse race there's no difference between AG observing the winner, and a MWI enthusiast imagining another winner in another world. AG
 
Each AG’s past at a particular trial is the same as all the other AGs at that trial, though each AG’s future diverges into further trees of different observed realities. The equal status of all branches means that your idea that AG observers on other branches are in some way different from the AG branch you happen to be on in that they only ever have one observation is just totally misconceived. To be blunt, you don’t get it.

What I get is that "ex hypothesi" covers a multiple of sins, including a complete disregard of experimental verification in physics. AG
 

Alan Grayson

unread,
Jan 15, 2021, 2:22:22 PM1/15/21
to Everything List
Why do you assume that the initial observer splits after initial trial when it's not observed? AG 

On Wednesday, January 6, 2021 at 5:28:51 AM UTC-7 Quentin Anciaux wrote:
Here a schema:
image.png

After 3 experiments, you have *8* worlds... each with the memory of the initial experiment, 4 of the 2nd version A and for of the 2nd version B... etc

Every *worlds* has a past which is linked directly with the previous experiment and to the initial experiment... in each world there is an ensemble of 3 results.

Quentin

Le mer. 6 janv. 2021 à 13:01, Alan Grayson <agrays...@gmail.com> a écrit :
I should have been more explicit; since the trials are independent, the other worlds implied by the MWI for any particular trial, are unrelated to the other worlds created for any OTHER particular trial. Thus, each other world has an ensemble with one element, insufficient for the existence of probabilities. AG

On Wednesday, January 6, 2021 at 4:41:57 AM UTC-7 Alan Grayson wrote:
On Wednesday, January 6, 2021 at 3:33:52 AM UTC-7 johnk...@gmail.com wrote:
On Tue, Jan 5, 2021 at 10:05 PM Alan Grayson <agrays...@gmail.com> wrote:

>> One world contains an Alan Grayson that sees the electron go left, another world is absolutely identical in every way except that it contains a  Alan Grayson that sees the electron go right. So you tell me, which of those 2 worlds is "THIS WORLD"?

> It's the world where a living being can observe the trials being measured. The other world is in your imagination (if you believe in the MWI). AG 

From that response I take it you have abandoned your attempt to poke logical holes in the Many Worlds Interpretation and instead have resorted to a pure emotional appeal; namely that there must be a fundamental law of physics that says anything Alan Grayson finds to be odd cannot exist, and Alan Grayson finds many Worlds to be odd. Personally I find Many Worlds to be odd too, although it's the least odd of all the quantum interpretations, however I don't think nature cares very much if you or I approve of it or not. From experimentation it's clear to me that if Many Worlds is not true then something even stranger is. 

I have no idea whatsoever, how you reached your conclusions above. There are things called laboratories, where physicists conduct experiments, some of which are quantum experiments with probabilistic outcomes. The world in which such things exist, I call THIS world. Worlds postulated to exist based on the claim that any possible measurement, must be a realized measurement in another world, I call OTHER worlds. Those OTHER worlds are imagined to exist based on the MWI. These are simple facts. I am not making any emotional appeals to anything. The possible oddness of the Cosmos is not affirmed or denied here. I agree the Cosmos might be odd, possibly very odd, but this has nothing to do with our discussion. The core of my argument is that since the trial outcomes in quantum experiments are independent of one another, there's no reason to claim that each of the OTHER worlds accumulates ensembles, as an ensemble is created in THIS world. Without ensembles in those OTHER worlds, the MWI fails to affirm the existence of probability in any of those OTHER worlds. AG 
 See my new list at  Extropolis

John K Clark


--
You received this message because you are subscribed to the Google Groups "Everything List" group.
To unsubscribe from this group and stop receiving emails from it, send an email to everything-li...@googlegroups.com.

John Clark

unread,
Jan 15, 2021, 3:18:53 PM1/15/21
to everyth...@googlegroups.com
On Fri, Jan 15, 2021 at 2:22 PM Alan Grayson <agrays...@gmail.com> wrote:

> Why do you assume that the initial observer splits after initial trial when it's not observed? AG 

For heaven sake haven't you been listening?! Because that is the least bizarre interpretation anybody can think of to explain the utterly bizarre results observed from the two slit experiment. There is just no getting around it, if Many Worlds isn't true then something even stranger must be. 

John K Clark

Alan Grayson

unread,
Jan 15, 2021, 5:34:03 PM1/15/21
to Everything List
Why not assume the wf applies only before the measurement? Or why not withhold judgement on a phenomenon not yet understood? Instead you totally dismiss empirical evidence that no one ever observes a split. AG

Alina Gutoreva

unread,
Jan 15, 2021, 6:27:58 PM1/15/21
to everyth...@googlegroups.com
"Before an idea is told, it’s never been true?”-kind of idea OR levels”-kind of idea? Or both? I’m confused.

Please, don’t be angry.

Ally
 

--
You received this message because you are subscribed to the Google Groups "Everything List" group.
To unsubscribe from this group and stop receiving emails from it, send an email to everything-li...@googlegroups.com.

Brent Meeker

unread,
Jan 15, 2021, 6:48:34 PM1/15/21
to everyth...@googlegroups.com
The Wigner's friend experiment proves a lot less than they headlines hype.



The "friends" are quantum 1-degree-of-freedom systems, so they don't decohere, and it's questionable to say they've measured anything.

Brent

smitra

unread,
Jan 15, 2021, 8:01:03 PM1/15/21
to everyth...@googlegroups.com
Decoherence should be irrelevant. Whether or not you (considered as some
given physical system) have measured something, should not only depend
on the entanglement between the measured system and those that belong to
you.

Saibal

On 16-01-2021 00:48, 'Brent Meeker' via Everything List wrote:
> The Wigner's friend experiment proves a lot less than they headlines
> hype.
>
> The "friends" are quantum 1-degree-of-freedom systems, so they don't
> decohere, and it's questionable to say they've measured anything.
>
> Brent
>
> On 1/15/2021 3:27 PM, Alina Gutoreva wrote:
>
>> "Before an idea is told, it’s never been true [1]?”-kind of
>> idea OR “levels [2]”-kind of idea? Or both [3]? I’m confused.
>>
>> Please, don’t be angry.
>>
>> Ally
>>
>> On 15 Jan 2021, at 22:34, Alan Grayson <agrays...@gmail.com>
>> wrote:
>> Why not assume the wf applies only before the measurement? Or why
>> not withhold judgement on a phenomenon not yet understood? Instead
>> you totally dismiss empirical evidence that no one ever observes a
>> split. AG
>>
>> On Friday, January 15, 2021 at 1:18:53 PM UTC-7 johnk...@gmail.com
>> [4] wrote:
>>
>> On Fri, Jan 15, 2021 at 2:22 PM Alan Grayson <agrays...@gmail.com>
>> wrote:
>>
>>> Why do you assume that the initial observer splits after initial
>> trial when it's not observed? AG
>>
>> For heaven sake haven't you been listening?! Because that is the
>> least bizarre interpretation anybody can think of to explain the
>> utterly bizarre results observed from the two slit experiment. There
>> is just no getting around it, if Many Worlds isn't true then
>> something even stranger must be.
>>
>> John K Clark
>
> --
> You received this message because you are subscribed to the Google
> Groups "Everything List" group.
> To unsubscribe from this group and stop receiving emails from it, send
> an email to everything-li...@googlegroups.com.
> To view this discussion on the web visit
> https://groups.google.com/d/msgid/everything-list/2a6bf029-a37a-4049-ab90-0ee889ba9820n%40googlegroups.com
> [5].
>
> --
> You received this message because you are subscribed to the Google
> Groups "Everything List" group.
> To unsubscribe from this group and stop receiving emails from it, send
> an email to everything-li...@googlegroups.com.
> To view this discussion on the web visit
> https://groups.google.com/d/msgid/everything-list/5BC6546E-8F24-4D9E-B432-23A92433AB94%40gmail.com
> [6].
>
> --
> You received this message because you are subscribed to the Google
> Groups "Everything List" group.
> To unsubscribe from this group and stop receiving emails from it, send
> an email to everything-li...@googlegroups.com.
> To view this discussion on the web visit
> https://groups.google.com/d/msgid/everything-list/0dc3eab3-583d-2c41-d7c7-9dfc8d4f9145%40verizon.net
> [7].
>
>
> Links:
> ------
> [1]
> https://www.technologyreview.com/2019/03/12/136684/a-quantum-experiment-suggests-theres-no-such-thing-as-objective-reality/
> [2] https://t.me/decision_insights/523
> [3] https://t.me/decision_insights/311
> [4] http://gmail.com
> [5]
> https://groups.google.com/d/msgid/everything-list/2a6bf029-a37a-4049-ab90-0ee889ba9820n%40googlegroups.com?utm_medium=email&amp;utm_source=footer
> [6]
> https://groups.google.com/d/msgid/everything-list/5BC6546E-8F24-4D9E-B432-23A92433AB94%40gmail.com?utm_medium=email&amp;utm_source=footer
> [7]
> https://groups.google.com/d/msgid/everything-list/0dc3eab3-583d-2c41-d7c7-9dfc8d4f9145%40verizon.net?utm_medium=email&utm_source=footer

Pierz Newton-John

unread,
Jan 15, 2021, 8:16:25 PM1/15/21
to everyth...@googlegroups.com
Bruce thinks that MWI violates Born’s rule and generally hates MWI. I don’t think his argument about the Born rule holds water, though it’s a well-known one. David Deutsch’s chapter on MWI in The Beginning of Infinity presents a refutation of that argument that I personally find convincing. But Bruce does at least understand MWI. I’m sure that while he shares your disdain for it, he would not accept that your specific argument against is valid. All you are doing is asserting that the other observers are imaginary, which is not a valid form of argument. 

Each AG’s past at a particular trial is the same as all the other AGs at that trial, though each AG’s future diverges into further trees of different observed realities. The equal status of all branches means that your idea that AG observers on other branches are in some way different from the AG branch you happen to be on in that they only ever have one observation is just totally misconceived. To be blunt, you don’t get it.

What I get is that "ex hypothesi" covers a multiple of sins, including a complete disregard of experimental verification in physics. AG

A hypothesis by definition has no experimental verification - yet! The same objection could have been made against General Relativity in 1915. But I’m not trying to convince you of MWI. Only that your argument about it not giving rise to probabilities is not valid. You can’t assume the hypothesis isn’t valid in your argument against it. You have to show how it’s logically inconsistent. 

 

As I previously indicated, these other AGs are disjoint from each other.
 
What do you mean by "disjoint" exactly? Mathematically "disjoint" means "having no elements in common". In the case of AG's who have measured different results, initially their worlds have only this difference between them, so in that sense they are not disjoint. If you mean they cannot interact with one another, and inhabit diverging realities, then that is only the case discounting interference, which we cannot do, because without interference effects we dot have quantum mechanics. This word disjoint seems to be central to your objection, but you need to define precisely what is meant by it or we cannot assess the validity of your claims.

The only way to remedy this situation is to add another postulate to your MWI. AG
 
No idea what we need to remedy. I'll ask my question again, adjusting it slightly. Does AG record a seemingly a random string of 1s and 0s in this experiment if MWI describes reality?

You seem to be assuming the other worlds created according to the MWI interact with other due to interference. Since these other worlds are never observed, I call them "imaginary"; and more important, no observations of interacting other worlds have ever been made, within QM or without QM. So the MWI is a huge stretch, at best. AG

Im not assuming it. It’s part of QM that wave functions interfere with themselves. In MWI that translates to different “worlds” interfering. That interference is extremely limited since it only occurs to the extent that two branches can become identical again after having diverged. Nonetheless it is the basis for proposed experimental proofs of MWI. One argument goes that if we can make a quantum computer with a sufficiently large number of qubits, we can prove the existence of other worlds because the other worlds are the only place we can get all that information from. So, no, this is not my assumption, it’s intrinsic to MWI. Again, if you don’t get that, you don’t get MWI. You can’t expect your arguments to be taken seriously if you don’t understand basics like this.

 

--
You received this message because you are subscribed to a topic in the Google Groups "Everything List" group.
To unsubscribe from this topic, visit https://groups.google.com/d/topic/everything-list/xsl8cSDT4M8/unsubscribe.
To unsubscribe from this group and all its topics, send an email to everything-li...@googlegroups.com.
To view this discussion on the web visit https://groups.google.com/d/msgid/everything-list/9028f9b4-f111-4366-bb84-f4024d15202do%40googlegroups.com.

--
You received this message because you are subscribed to a topic in the Google Groups "Everything List" group.
To unsubscribe from this topic, visit https://groups.google.com/d/topic/everything-list/xsl8cSDT4M8/unsubscribe.
To unsubscribe from this group and all its topics, send an email to everything-li...@googlegroups.com.

smitra

unread,
Jan 15, 2021, 8:39:26 PM1/15/21
to everyth...@googlegroups.com
On 16-01-2021 02:00, smitra wrote:
> Decoherence should be irrelevant. Whether or not you (considered as
> some given physical system) have measured something, should not only
> depend on the entanglement between the measured system and those that
> belong to you.

Correction: ...Should only depend on..

Saibal
It is loading more messages.
0 new messages